0% found this document useful (0 votes)
199 views93 pages

Surgery 2 PDF

Uploaded by

BALAJI GKRR
Copyright
© © All Rights Reserved
We take content rights seriously. If you suspect this is your content, claim it here.
Available Formats
Download as PDF, TXT or read online on Scribd
0% found this document useful (0 votes)
199 views93 pages

Surgery 2 PDF

Uploaded by

BALAJI GKRR
Copyright
© © All Rights Reserved
We take content rights seriously. If you suspect this is your content, claim it here.
Available Formats
Download as PDF, TXT or read online on Scribd
You are on page 1/ 93

Copyright © 2014 Delhi Academy of Medical Sciences, All Rights Reserved.

1/93
Test Information
Test Name SWTS-SURGERY-2 2017(MDMS) Total Questions 200

Test Type Examination Difficulty Level Difficult

Total Marks 600 Duration 120minutes

Test Question Language:- ENGLISH

(1). The earliest manifestation seen after Gastrectomy :

a. Incidence of infection

b. Loss of storage capacity

c. Loss of HCL

d. Loss of intrinsic factor.

Solution. (b) Loss of storage capacity


Ref: Bailey -1074
Sol :

Correct Answer. b

(2). Anastomosis of small bowel is done by :

a. Suture is one layer by nonabsorbable suture

b. Suture in two layer by nonabsorbable suture

c. Suture in two layer by absorbable suture

d. Suture in layers by absorbable suture.

Solution. (c) Suture in two layer by absorbable suture


Ref: Schwartz - 1072
Sol :

Correct Answer. c

Copyright © 2014 Delhi Academy of Medical Sciences, All Rights Reserved. 2/93
(3). A 56-year-old woman has not passed stools for the last 14 days. X-ray shows no air fluid levels, probable diagnosis is :

a. Paralytic ileus

b. Aganglionosis of the colon

c. Intestinal pseudo-obstruction

d. Duodenal obstruction

Solution. (c) Intestinal pseudo-obstruction


Ref: Harrison - 1912
Sol : INTESTINAL PSEUDO-OBSTRUCTION.
- Abdominal X-ray shows gaseous distension of large and small bowel, presence of fluid level is unusual and raise possibility of
mechanical obstruction.
- Usually seen in elderly and may be associated with pneumonia, myocardial infarction, chronic myxoedema.
- Abdominal distention along with features of obstruction (failure to pass feces and flatus) are main symptoms.
- Rectal examination reveals some faeces (contrary to mechanical obstruction in which rectum is empty).
- Treatment is conservative, provided acute abdomen is ruled out.

Correct Answer. c

(4). The highest percentage of colon cancers are found in the

a. Cecum

b. Ascending colon

c. Descending colon

d. Rectosigmoid

Solution. (d) Rectosigmoid


Ref: Read the text below
Sol :
- The most common site of colorectal cancer is the sigmoid colon (25%) followed by the rectum (21%) followed by cecum (20%) followed
by rectosigmoid junction (20%) followed by transverse colon (15%) followed by ascending colon (10%).

Correct Answer. d

(5). Paralytic ileus is seen in all of the following except :

a. Spinal cord injury

b. Hypocalcemia

c. Hypermagnesemia

d. Uremia

Solution. (c) Hypermagnesemia


Ref: Bailey - 1201
Sol : Paralytic ileus (neurogenic ileus) : Defined as failure of transmission of peristaltic wave secondary to neuromuscular failure
CAUSES
- Peritonitis (MC)
- Reflex ileus : Following fracture of spine or ribs, retroperitoneal hemorrhage.
- Infection : Intra-abdominal abscess, pneumonia, sepsis.
- Metabolic : Uremia, hypokemia are most common contributing factors, hyponatremia. Hypo and hypermagnesemia, hypothyroidism can
also contribute.
- Ischemia : Mesenteric ischemia, myocardial infarction. ]
- Postoperative : Paralytic ileus should be considered only after 72 hrs. of operation.
- Drugs : Anticholinergics, opiates, calcium channel blockers, tricyclic antidepressants.

Correct Answer. c

Copyright © 2014 Delhi Academy of Medical Sciences, All Rights Reserved. 3/93
(6). Commonest cause of neonatal intestinal obstruction :

a. Duodena atresia

b. Jejunal atresia

c. Necrotizing enterocolitis

d. Meconium ileus

Solution. (a) Duodena atresia


Ref: Bailey - 1199
Sol :
- Congenital atresia is not common cause of neonatal intestinal obstruction, out of which duodenal atresia is most leading one. Duodenal
atresia
- Atresia at the level of ampulla of Vater. 30% babies have associated Down’s syndrome.
- Radiograph shows double bubble sign with two air fluid level.
- Treatment – Immediate surgery after resuscitation. Duodeno jejunostomy is the operation of choice.

Correct Answer. a

(7). Most common benign tumour of the stomach is -

a. Adenoma

b. Lipoma

c. Hamartoma

d. Leiomyoma

Solution. (a) Adenoma


Ref: Read the text below
Sol :
- Epithelial polyps are most common benign tumor of stomach. They are of five types.
- Hyperplastic (most common) type.
- Adenomatous (2nd most common).
- Hamartmatous.
- Inflammatory.
- Hetrotopic.

Correct Answer. a

(8). The operation where in the stump of the stomach is directly anastomosed to the stump of the duodenum is called –

a. Polya’s gastrectomy

b. Hoffrneister gastrectomy

c. Billroth I gastrectomy

d. Billroth II gastrectomy

Solution. (a) Billroth I gastrectomy


Ref: Read the text below
Sol :
- In Bilroth type 1 gastrectomy the lower half of stomach is removed and the cut stomach is anastomosed to first part of duodenum.
- In type II two third of stomach is removed, duodenal stump is closed and stomach is anastomosed to the jejunum.

Correct Answer. a

Copyright © 2014 Delhi Academy of Medical Sciences, All Rights Reserved. 4/93
(9). Best selective laser for the ablation of prostate is: -

a. Holmium

b. CO2

c. KTP

d. Nd:YAG

Solution. (c) KTP Sol: LASER ABLATION Best Source


- KTP Laser
Other Options
- Nd:YAG laser
- Visual laser ablation of the prostate (VLAP)
- Contact laser prostatectomy
- Interstitial laser prostatectomy (ILP)
Holmium laser prostatectomy

Correct Answer. c

(10). A 23 year male presents with a grossly contaminated wound on his right thigh. Which of the following are effective in cleansing the
wound and reducing the risk of infection?

a. Low pressure saline irrigation (e.g. NS bag under gravity).

b. Irigation with povidone-iodine.

c. Wound scrubbing.

d. High pressure (5-8 psi) saline irrigation.

Solution. (d) High pressure (5-8 psi) saline irrigation


Reference – Read the text below
Sol:
- Effective wound irrigation reduces the risk of wound infection.
- However irrigation pressures of five to eight PSI are required, this can be achieved with and 19 gauge needle and 35 ml syringe. Low
pressure irrigation such as his achieved by an IV giving set is ineffective.
- Scrubbing of wounds traumatizes tissues and may increase the risk of infection.
- Antiseptics should not be used within wounds as they can be cytotoxic and may increase the risk of wound infection.

Correct Answer. d

(11). Ectopic gastric mucosa is a feature of ?

a. Jejunal diverticulitis

b. Cecal diverticulitis

c. Sigmoid diverticulitis

d. Meckels diverticulitis

Solution. (d) Meckels diverticulitis


Ref– Read the text below
Sol:
- Meckel's diverticulum is the most common congenital anomaly of the gastrointestinal tract with a commonly accepted incidence of 2%
in the general population.
- Of these, about 57% contain ectopic gastric mucosa which is responsible for the clinical symptoms. Radionuclide imaging is a primary
study for the detection of the ectopic gastric mucosa in a Meckel's diverticulum.
- Using Tc-99m pertechnetate, which is actively accumulated and secreted by the mucoid cells of the gastric mucosa, there is
approximately a 90% detection rate in children and lower in adults.

Correct Answer. d

Copyright © 2014 Delhi Academy of Medical Sciences, All Rights Reserved. 5/93
(12). A 65-year-old man is referred with abnormal liver function and undergoes a liver biopsy. Which of the following count against hepatic
cirrhosis?

a. Fibrous septa formation

b. Granuloma formation

c. Liver cell necrosis

d. Nodular regeneration

Solution. (b) Granuloma formation


Reference – Read the text below
Sol:
- Granuloma formation is not classically seen in cirrhosis, which can be micro or macronodular in type.
- In the micronodular form,the nodules are less than 3mm across with uniform liver involvement - seen in alcohol or biliary disease.
-In the macronodular form, there are larger nodules, classically seen in chronic viral hepatitis.

Correct Answer. b

(13). Contrecoup does not produce :

a. Cerebral laceration

b. Extradural haematoma

c. Subdural haematoma

d. Intracerebral haematoma

Solution. (b) Extradural haematoma


Ref: Read the text below.
Sol:
- Contrecoup lesions – means that the lesion is present in an area opposite the side of impact, chiefly due to local distortion of skull and
sudden rotation of the head resulting from blow, which cause shear strains due to the pulling apart of the constituent particle of the
brain.
- A blow to the head produces coup contusions while contrecoup contusions are either small or absent. A fall on the head produces
contrecoup contusion while coup contusions are small or absent.
- Extradural hemorrhage – The hematoma cannot be contrecoup unless the skull has been grossly deformed.
- Subdural hemorrhage – The bridging veins over the upper and posterior aspect of the cerebral hemisphere are under the greatest
strains in rotational movements of the head.

Correct Answer. b

(14)
Vicryl, the commonly used suture material is a :
.

a. Homopolymer of polydioxanone

b. Copolymer of glycolide and lectide

c. Homopolymer of glycoside

d. Homopolymer of lectide

Solution. (b) Copolymer of glycolide and lectide


Ref: Bailey and Loves short Practice of Surgery --99
Sol:

Correct Answer. b

Copyright © 2014 Delhi Academy of Medical Sciences, All Rights Reserved. 6/93
(15) During investigation of hydronephrosis, isotope renogram is useful
. mainly in :

a. Detecting vesicoureteric reflux

b. Anatomical definition

c. Distinguishing between non-obstructed and obstructed system

d. Identifying ectopic kidney tissue

Solution. (c) Distinguishing between non-obstructed and obstructed


system
Ref: Bailey and Love’s Short Practice of Surgery--1314
Sol:Imaging of hydronephrosis :

Correct Answer. c

(16). Association of carcinoma gall bladder with gall stones is about

a. 50%

b. 70%

c. 90%

d. 20%

Solution. (c) 90%


Ref: Davidson’s – 993
Sol:
- “In over 95% of patients with gall bladder cancer, there are accompanying gallstones”. 90% are adenocarcinomas.
- Presents usually as benign disease (gallstones).
- CA 19-9 is elevated in 80% of patients.
- Prognosis is poor-95% mortality in 1 year.

Correct Answer. c

Copyright © 2014 Delhi Academy of Medical Sciences, All Rights Reserved. 7/93
(17). For a Non seminomatous tumour with high bulk retroperitoneal node involvement best treatment is?

a. Inguinal orchidectomy and Radiotherapy

b. Inguinal orchidectomy and RPLND

c. Inguinal orchiedectomy and Chemotherapy

d. Inguinal orchidectomy and Chemotherapy followed by RPLND for residual disease

Solution. (d) Inguinal orchidectomy and Chemotherapy followed by RPLND for residual disease
Ref: Read the text below
Sol:

Correct Answer. d

(18). Maximum time till testis can descend is?

a. 4 months

b. 6 months

c. 9 months

d. 12 months

Solution. (c) 9 months


Ref: Read the text below
Sol: Undescended testis (UDT) is seen in 3% of full term babies, 30% of premature babies but at the age of 9 months it is equal for both
(0.8%)

Correct Answer. c

Copyright © 2014 Delhi Academy of Medical Sciences, All Rights Reserved. 8/93
(19). Shown deformity in IVP is seen in

a. Ureteric TB

b. Retroperitonela fibrosis

c. Medullary sponge kidney

d. Rtero-caval ureter

Solution. (b) Retroperitonela fibrosis


Ref: Read the text below
Sol: Retroperitoneal fibrosis
- Fibrotic plaque centered over l4- l5 „ near sacral promontory and extending from renal hilum to sacral promontory and outer borders of
psoas.
- Prevalence 1 in 2 lacs
- Male > female 2 :1 (30 - 60 years)
Causes: 2/3 idiopathic (ormond's disease) Known causes:
1) Drugs- methysergide methyldopa, lsd other ergots (eg. Bromocriptine) phenacetin amphetamines β- blockers
2) Malignancies - lymphomas, sarcomas
3) Inflammatory conditions. Ibd, diverticulitis
4) Radiation
Presentation Obstructive uropathy (earliest + mc- Ureter involvemnet)
Diagnosis
Ivporrgp:
1. Medial pulling of ureters
2. Pipestem ureters ct/mri: delineate rp fibrosis
Medical Tt- Tamoxifen, Aziathioprine, Pencillamine, Steroid Surgical- Intraperitonealization (ie, surgical rerouting of ureters into the
peritoneal cavity) of the ureters is sometimes used as a therapeutic intervention to free the ureters from the fibrotic plaque and relieve
associated obstruction.

Correct Answer. b

Copyright © 2014 Delhi Academy of Medical Sciences, All Rights Reserved. 9/93
(20). Cardinal features of Intestinal obstruction are as follows, except for

a. Abdominal pain

b. Vomiting

c. Diarrhea

d. Distension of Abdomen

Solution. (c) Diarrhea


Ref: Bailey and Love- 1193.
Sol: Classic features of acute intestinal obstruction include :
- Intestinal colic : colicky central abdominal pain.
- Vomiting : early in high obstruction.
- Abdominal distension : extent depends on level of obstruction.
- Absolute constipation/obstipation : late feature of small bowel obstruction.
- Dehydration : associated with tachycardia, hypotension and oliguria.
- Features of peritonism indicate strangulation or perforation.

Correct Answer. c

(21). Chronic Urethral Obstruction due to benign prostatic hypertrophy can lead to the following change in the kidney parenchyma

a. Hyperplasia

b. Hypertrophy

c. Atrophy

d. Dysplasia

Solution. (c) Atrophy


Ref: Robbin’s - 1013.
Sol:
- “Chronic urethral obstruction due to BPH leads to hydronephrosis.
- In chronic cases, the kidney shows cortical tubular atrophy with marked interstitial fibrosis.
- Progressive blunting of the calyces occurs and these eventually become cupped.”

Correct Answer. c

(22). The most common position adopted for surgical procedures is

a. Trendelenburg position

b. Lithotomy position

c. Supine position

d. Prone position

Solution. (c) Supine position


Ref: Basic of Anesthesia- 292
Sol:
- “The supine position is the most common surgical position and produces the least hemodynamic and ventilator changes.”

Correct Answer. c

Copyright © 2014 Delhi Academy of Medical Sciences, All Rights Reserved. 10/93
(23). Lithotripsy is the equipment used to crush the stones

a. High frequency (CW) US is used

b. High frequency pulsed wave is used

c. Low frequency continuous wave is used

d. Medium range (2-5 MHz) pulsed wave are used

Solution. (a) High frequency (CW) US used


Ref: Read the text below
Sol:
- In percutaneous ultrasound lithotripsy, the ultrasound probe emits high frequency ultrasound energy that has a simple drilling effect
upon direct exposure to the stone.

Correct Answer. a

(24). Commonest type of mesenteric cyst is

a. Enterogenous

b. Chylolymphatic

c. Dermoid

d. Urogenital remnant

Solution. (b) Chylolymphatic


Ref: Bailey and Love’s - 1005.
Sol: Mesenteric cysts are classified as
- Chylolymphatic,
- Simple (mesothelial)
- Enterogenous
- Urogenital remnant
- Dermoid (teratomatous cyst)
- Chylolymphatc cyst, the most common mesenteric cyst, arises in congenitally misplaced lymphatic tissue that has no efferent
communication with he lymphatic system; it arises most frequently in the mesentery of the ileum usually solitary has a blood supply that
is independent from the adjacent intestine so that enucleation is possible without resection of intestine.

Correct Answer. b

(25). A 15 year old boy is admitted with a history and physical findings consistent with appendicitis. Which of the following findings is most
likely to be positive ?

a. Pelvic Crepitus

b. Iliopsoas sign

c. Murphy’s Sign

d. Flank ecchymosis

Solution. (b) Iliopsoas sign


Ref: Bailey and Love’s- 1208,
Sol:
- Psoas sign/Iloposas sign: Occasionally an inflamed appendix lies on the psoas muscle, and the patient, often a young adult will lie with
the right hip flexed for pain relief.
- Pain occurs when the patient tries to raise the leg up against the hand of the examiner pushing down against the leg above the knee.

Correct Answer. b

Copyright © 2014 Delhi Academy of Medical Sciences, All Rights Reserved. 11/93
(26). Commonest intra-abdominal tumor below 2 years of age :

a. Neuroblastoma

b. Wilm’s tumor

c. Hepatoblastoma

d. Lymphoma

Solution. (a) Neuroblastoma


Ref: OP Ghai, 7th edn. Pg. 590, 591.
Sol:
- Neuroblastoma is the most common intraabdominal solid tumor in childhood.
- It is of embryonic origin, arising from primitive sympathetic system cells derived from the neural crest
- MC arises from the adrenal glands (30%).
- 90% arise within 5 years of age and 50% within 2 years; chromosome 1p; n-myc oncogene.
- Associated diseases : Hirschsprung’s disease, heterochromia, fetal hydantoin and fetal alcohol syndromes, and Friedrich ataxia.
- Metastasis is to the facial skeleton and orbits. Gold standard for diagnosis is examination of tumor tissue by histopathology and
immunohistochemistry.

Correct Answer. a

(27). All of the following are indications for common bile duct (CBD) exploration except :

a. Palpable stone in CBD

b. Dilated CBD

c. Presence of jaundice

d. Multiple stones in the gallbladder

Solution. (d) Multiple stones in the gallbladder


Ref:Bailey and Love’s Short Practice of Surgery1124.
Sol: Indications for choledochotomy :
- Palpable duct stones
- There is jaundice or a history of jaundice or cholangitis
- The common bile duct is dilated
- Liver function tests are abnormal, in particular raised alkaline phosphatase

Correct Answer. d

Copyright © 2014 Delhi Academy of Medical Sciences, All Rights Reserved. 12/93
(28). In a patient on anticoagulant therapy, the INR is maintained at :

a. 1.5 to 2.5 times the normal

b. 2.5 to 3.5 times the normal

c. 3.5 to 4.5 times the normal

d. 4.5 to 5.5 times the normal

Solution. (b) 2.5 to 3.5 times the normal


Ref: Read the text below
Sol:
- The lab control of heparin is a PTT/clotting time (desirable) and for warfarin is PT/INR (essential).
-Dose regulation of oral anticoagulant – A standardized system called international normalized ration (INR) based on the use of human brain
Tp has been developed by WHO.

Correct Answer. b

(29). Acute compartment syndrome in a limb is characterized by :

a. Severe pain, marked muscle tenderness and distal sensory disturbances

b. Marked muscle tenderness and severe pain

c. Distal sensory disturbances and marked muscle tenderness

d. Severe pain and distal sensory disturbances

Solution. (a) Severe pain, marked muscle tenderness and distal sensory disturbances
Ref:Read the text below
Sol:
- Compartment syndrome : Injuries with a high risk of developing compartment syndrome are : Supracondylar fracture. Forearm bone
fracture. Closed tibial fracture. Crush injuries to leg and forearm.
- An excessive pain, not relived by usual doses of analgesics. Stretch test is the earliest sign of impending compartment syndrome.
- Other signs tens tense compartment (pressure > 40mm water is indicative), hypoaesthesia in distribution of involved nerve, muscle
weakness. Nerve damage may result in motor and sensory loss.

Correct Answer. a

Copyright © 2014 Delhi Academy of Medical Sciences, All Rights Reserved. 13/93
(30). For replantation surgery, the detached digit or limb is best preserved in cold:

a. Glycerol

b. Distilled water

c. Hypertonic saline

d. Isotonic saline

Solution. (d) Isotonic saline


Ref: Bailey and Love’s Short Practice of Surgery- 524
Sol: Replantation surgery
- Indication – (a) Thumb, (b) Single digit in children, (c) Single digit at distal interphalangeal joint level, (d) multiple digits, (e) hands,
wrist, forearm.
- Relative contraindications – (a) Single digits in adult, (b) crushed, mangled or avulsed part, (c) Poor condition, (d) long warm ischaemic
time.
Amputed part should be wrapped in a sterile cloth soaked in sterile saline and placed in plastic bag placed in water and crushed ice.

Correct Answer. d

(31). In chest wall reconstruction, the preferred flap is :

a. Local random pattern skin flap

b. Pedicled muscle base flap

c. Axial pattern flap

d. Split skin graft

Solution. (b) Pedicled muscle base flap


Ref:Read the text below
Sol:
- Based on thoracoacromial supply, pectoralis major flap, the second largest and has become flap of choice for sterna defect.

Correct Answer. b

(32). A 25- year-old male is admitted with blunt injury chest. On examination, patient if found to have significant flail chest. The choice of
immediate treatment is :

a. Intercostal drainage

b. Tracheal intubation and positive pressure ventilation

c. Aspiration

d. Strapping of chest wall

Solution. (b) Tracheal intubation and positive pressure ventilation


Ref:Bailey and Love’s Short Practice of Surgery 24th --868
Sol:
- Flail chest – creates a right to left shunt and prevents full saturation of arterial blood.
- If flail segment is small and not embarrassing respiration – Nursed on a high – dependency unit with regular blood gas analysis and
good analgesia until flail segment stabilizes.
- In more severe case – endotracheal intubation with positive pressure ventilation for up to 3 weeks until the fracture becomes less
mobile.

Correct Answer. b

Copyright © 2014 Delhi Academy of Medical Sciences, All Rights Reserved. 14/93
(33). Widened anionic gap is not seen in:

a. Acute renal failure

b. Diarrhea

c. Lactic acidosis

d. Diabetic ketoacidosis

Solution. (b) Diarrhea


Ref.: Read the text below
Sol :
- In diarrhea anion gap is normal
Widened anionic gap
- Acute renal failure
- Lactic acidosis
- Diabetic ketoacidosis

Correct Answer. b

(34). True colour of cholesterol stone is?

a. Black

b. Brown

c. Dark yellow

d. Pale yellow

Solution. (d) Pale yellow


Ref.: Read the text below
Sol :

Correct Answer. d

Copyright © 2014 Delhi Academy of Medical Sciences, All Rights Reserved. 15/93
(35). Pseudo obstruction of intestine is also known as?

a. Hartmann’s syndrome

b. Ozili’s syndrome

c. Ogilive’s syndrome

d. Merizzi syndrome

Solution. (c) Ogilive’s syndrome


Ref.: Read the text below
Sol :
- Ogilvie syndrome is the acute obstruction and dilation of the colon in the absence of any mechanical obstruction in severely ill patients.
- Colonic pseudo-obstruction is characterized by massive dilatation of the cecum(diameter > 10 cm) and right colon on abdominal X-ray.
- It is a type ofmegacolon, sometimes referred to as "acute megacolon," to distinguish it fromtoxic megacolon.

Correct Answer. c

(36). The ideal management for squamous cell carcinoma of middle one-third of lower lip (1x1 cm size) without lymph node involvement would
be:

a. Chemoradiotherapy

b. Local resection followed by radiotherapy

c. Excision with primary repair

d. Primary irradiation

Solution. (c) Excision with primary repair


Ref.: Chassin’s Operative Surgery in General Surgery.
Sol : Carcinoma of the lip:
- Both surgery and radiotherapy are frequently employed and highly effective methods of treatment, each giving cure rates of about 90%.
- Up to one-third of lower lip can be removed by a shaped excision with primary closure.
- This method is suitable for tumors of up to 2 cm in diameter.
- If more than one-third of the lip is removed, primary closure results in microstomia.
- Therefore, for more extensive lip resection, it is necessary to utilize local flaps for reconstruction.
- For large central defects of the lower lip, particularly in patients who do not have, wrinkled faces, the stepladder’ approach of johanson
gives excellent consumesis.

Correct Answer. c

(37). The most common cause of death in thermal burns is:

a. Convulsion

b. Aspiration pneumonia

c. Hypovolemic shock

d. Arrhythmias

Solution. (c) Hypovolemic shock


Ref.: Schwartz’s Principles of Surgery,
Sol :
- The most common cause of death in thermal burns is hypovolemic shock
- Bacteremia and bacteremic shock are the second commonest cause of death in burn following oligaemic shock.

Correct Answer. c

Copyright © 2014 Delhi Academy of Medical Sciences, All Rights Reserved. 16/93
(38). A patient has recurrent abdominal pain and jaundice. The blood investigation reveal reticulocytosis and hyperbilirubinaemia. What is the clinical
diagnosis?

a. Hereditary spherocytosis

b. Mirizzi’s syndrome

c. Choledochal cyst

d. Sclerosing cholangitis

Solution. (a) Hereditary spherocytosis


Ref.: Robbin’s Pathologic Basis of Disease- 625
Sol :

Correct Answer. a

(39). Which one of the following organism is not a causative agent for gas gangrene?

a. Clostridium difficile

b. Clostridium septicum

c. Clostridium novyi

d. Clostridium parfringens

Solution. (a) Clostridium difficile


Ref.: Aanathnarayan R. Texbook of Microbiology, 6th ed. Pg. 236
Sol : Gas producing clostridia :
- Cl. Perfringens (Cl. Welchii) – most common
- Cl. Novyi
- Cl. Septicum
- Cl. histolyticum

Correct Answer. a

Copyright © 2014 Delhi Academy of Medical Sciences, All Rights Reserved. 17/93
(40). A 17-year-old man is stabbed in the axilla. He is noted to have loss of function of the ulnar nerve. If nonoperative therapy is used for
treatment of this nerve injury, at what rate is the nerve expected to regenerate?

a. 1 mm/wk

b. 1 mm/mo

c. 1 mm/d

d. 10 mm/mo

Solution. (c) 1 mm/d


Ref– Read the text below
Sol:
- When a nerve axon is disrupted, such as occurs with axonotmesis or neurotmesis, the distal nerve degenerates to the motor endplate
and the proximal nerve degenerates back to the nodes of Ranvier.
- If scar tissue does not prohibit renewal, nerves will regenerate at an average of 1 mm/d.

Correct Answer. c

(41). A 72 year old dairy farmer presents with a small white lesion on his left cheek.It has grown slowly over several months, and it now has a
central ulcer.What is the most likely nature of the lesion?

a. Amelanotic malignant melanoma

b. Basal cell carcinoma

c. Basal cell papilloma

d. None

Solution. (b) Basal cell carcinoma


Ref– Read the text below
Sol:
- This is because the appearances and location of this lesion are typical of basal cell carcinoma.
- It is a lesion associated with sun exposure, which is common in farmers.

Correct Answer. b

(42). A 43 year old male presents with weight loss and watery diarrhoea. Investigations reveal hypokalaemia with a pancreatic mass. Which of
the following would support the diagnosis of a VIPoma?

a. Achlorhydria

b. Hypoglycaemia

c. Increased Pancreatic polypeptide

d. Migratory erythema

Solution. (a) Achlorhydria


Reference – Read the text below
Sol:
- Achlorhydria is classically associated with VIPoma together with profuse diarrhoea, a hypokalaemic acidosis and hyperglycaemia.
- Migratory erythema is associated with a glucagonoma.
- Although raised pancreatic polypeptide is seen with a VIPoma it is unusual and is more commonly associated with its own syndrome.
- Pellagra is associated with the carcinoid syndrome.

Correct Answer. a

Copyright © 2014 Delhi Academy of Medical Sciences, All Rights Reserved. 18/93
(43). The National Triage Scale is an agreed method of triage for emergency department patients. A patient categorized as NTS 4 should be seen by a
medical officer?

a. Within 30 minutes.

b. Within 45 minutes.

c. Within 60 minutes.

d. Within 80 minutes.

Solution. (c) Within 60 minutes


Reference – Read the text below
Sol:

Correct Answer. c

(44). Best treatment of Bleeding type I gastric ulcer is?

a. Wedge resection of the ulcer

b. Oversewing the bleeding vessel

c. Distal gastrectomy

d. Distal gastrectomy and Truncal vagotomy

Solution. (c) Distal gastrectomy


Ref:Read the text below
Sol:
- For bleeding type 1 gastric ulcers, a distal gastrectomy with Billroth I anastomosis is usually performed
- For types 2 and 3 gastric ulcers, distal gastrectomy in combination with vagotomy is indicated.

Correct Answer. c

(45). A 50-year-old patient presents with symptomatic nephrolithiasis.He reports that he underwent a jejunoileal bypass for morbid obesity
when he was 39. One would expect to find

a. Pseudohyperparathyroidism

b. Hyperuric aciduria

c. Hyperoxaluria

d. Sporadic unicameral bone cysts

Solution. (c) Hyperoxaluria


Ref:Read the text below
Sol:
- Any patient who has lost much of the ileum (whether from injury, disease, or elective surgery) is at high risk of developing enteric
hyperoxaluria if the colon remains intact.
- Calcium oxalate stones will develop in at least 10% of these patients.
- The condition results from excessive absorption of oxalate from the colon through two related synergistic mechanisms: unabsorbed fatty
acids combine with calcium, which prevents the formation of insoluble calcium oxalate and allows oxalate to remain available for colonic
absorption; and unabsorbed fatty acids and bile acids also increase the permeability of the colon to the oxalate.

Correct Answer. c

Copyright © 2014 Delhi Academy of Medical Sciences, All Rights Reserved. 19/93
(46). In ulcerative coilitis with toxic megacolon lowest recurrence is seen in

a. Complete proctocolectomy and Brook's ileostomy

b. Ileo rectal anastomoses

c. Kock's pouch

d. Ileo anal pull through procedure

Solution. (a) Complete proctocolectomy and Brook's ileostomy


Ref:Read the text below
Sol:
- Total proctocolectomy with brooke's ileostomy removes almost all of the diseased segment.

Correct Answer. a

(47). The most appropriate therapy for the patient suffering from metabolic alkalosis with respiratory compensation would be

a. Infusion of 0.9% NaC1 with supplemental KC1 until clinical signs of volume depletion are eliminated

b. Infusion of isotonic (0.15 N) HC1 via a central venous catheter

c. Clamping the nasogastric tube to prevent further acid losses

d. Administration of acetazolamide to promote renal excretion of bicarbonate

Solution. (a) Infusion of 0.9% NaC1 with supplemental KC1 until clinical signs of volume depletion are eliminated
Ref:Read the text below
Sol:
- The development of a clinically significant metabolic alkalosis in a patient requires not only the loss of acid or addition of alkali, but
renal responses that maintain the alkalosis.
- The normal kidney can tremendously augment its excretion of acid or alkali in response to changes in ingested load.
- However, in the presence of significant volume depletion and consequent excessive salt and water retention, the tubular maximum for
bicarbonate reabsorption is increased.
- Correction of volume depletion alone is usually sufficient to correct the alkalosis, since the kidney will then excrete the excess
bicarbonate.

Correct Answer. a

(48). What is the most common complication after esophagectomy

a. Arrythmia

b. Pulmonary Collapse

c. Recurrent laryngeal nerve injury

d. Massive bleeding

Solution. (b) Pulmonary Collapse


Ref:Read the text below
Sol:
- Most common complication after esophagectomy is Pulmonary Collapse

Correct Answer. b

Copyright © 2014 Delhi Academy of Medical Sciences, All Rights Reserved. 20/93
(49). During a neck dissection, the styloid process is used as a landmark. Which of the following statements correctly pertains to one of the
four structures that attach to the styloid process?

a. The stylohyoid muscle attaches to the lesser horn of the hyoid bone

b. The styloglossus muscle acts to protrude the tongue

c. The stylohyoid ligament attaches to the lingula of the mandible

d. Distally the stylopharyngeus muscle is split by the digastric muscle

Solution. (a) The stylohyoid muscle attaches to the lesser horn of the hyoid bone
Ref:Read the text below
Sol:
- The stylohyoid muscle inserts onto the lesser horn of the hyoid bone (both derivatives of the second branchial arch) and raises that bone
during swallowing.
- The distal tendon of the stylohyoid muscle is split by the digastric muscle passing through its trochlea attached to the lesser horn.
- The styloglossus muscle acts to retract the tongue. The sphenomandibular ligament inserts onto the lingula of the mandibular foramen;
the stylohyoid ligament inserts onto the lesser horn of the hyoid bone.
- The stylopharyngeus muscle inserts onto the thyroid cartilage and into the middle pharyngeal constrictor.

Correct Answer. a

(50). The transverse rectus abdominis myocutaneous (TRAM) flap consists of a skin paddle based on the underlying rectus abdominis muscle,
which is supplied by vessels from the : -

a. Inferior epigastric artery

b. Superior epigastric artery

c. Superior pancreaticoduodenal artery

d. Inferior pacreaticoduodenal artery

Solution. (a)Inferior epigastric artery


Sol: Reconstruction of the breast and chest wall. Many different types of myocutaneous flaps are employed for breast reconstruction, but
the latissimus dorsi and the rectus abdominus myocutaneous flaps are most frequently used.
1. The latissimus dorsi myocutaneous flaps consists of a skin paddle based on the underlying latissimus dorsi muscle, which is supplied
by the thoracodorsal artery with contributions from the posterior intercostal arteries.
2. The transverse rectus abdominis myocutaneous (TRAM) flap consists of a skin paddle based on the underlying rectus abdominis
muscle, which is supplied by vessels from the deep inferior epigastric artery.
3. The free TRAM flap uses microvascular anastomoses to establish blood supply to the flap.

Correct Answer. a

(51). A 40-year-old man presented with a flat 1 cm x 1 cm scaly, itchy black mole on the front of thigh. Examination did not reveal any inguinal
lymphadenopathy. The best course of management would be:

a. FNAC of the lesion

b. Incision biopsy

c. Excisional biopsy

d. Wide excision with inguinal lymphadenectomy

Solution. (c) Excisional biopsy


Ref.: Cecil Textbook of Medicine- 1249.
Sol :
- Any skin lesion suspicious for melanoma should be biopsied with complete excision, including 1-2 mm margin of normal skin and
subcutaneous fat.
- An incisional biopsy may be necessary for lesions too large for complete excision.

Correct Answer. c

Copyright © 2014 Delhi Academy of Medical Sciences, All Rights Reserved. 21/93
(52). Vascular complications of acute pancreatitis include the following except

a. Splenic vein thrombosis

b. Splenic artery aneurysm

c. Gastroduodenal artery aneurysm

d. Middle colic artery thrombosis

Solution. (d) Middle colic artery thrombosis


Ref.: Harrisons’ Principles of Internal Medicine- 1902
Sol :
- Pseudoaneurysms – develop in up to 10% of patients with acute pancreatitis
- Splenic artery most frequently involved followed by inferior and superior pancreatic duodenal arteries.
- Diagnosis should be suspected in patients with pancreatitis having upper GI bleeding without an obvious cause.
- CT reveals contrast – enhanced lesion within or adjacent to suspected pseudocyst.
- Arteriography is necessary to confirm the diagnosis.

Correct Answer. d

(53). Ranson’s scoring system is used to predict severity of :

a. Acute pancreatitis

b. Chronic pancreatitis

c. Acute Cholecystitis

d. Acute Appendicitis

Solution. (a) Acute pancreatitis


Ref.: Read the text below
Sol : Ranson criteria for severity of acute pancreatitis At Admission
- Age > 55 years
- White cell count > 16.0 × 10°/L
-Blood glucose > 200 mg/dl
- Serum lactate dehydorgenase (LDH) > 350 IU/L
- Aspartate transaminase (SGOT) > 250 U/L
During Initial 48 Hours :
- Hematocrit decrease > 10 percentage points
- Blood urea nitrogen (BUN) incease > 5 mg/dl as urea
- PaO2 < 60 mm Hg
- Base deficit > 4 mEq/L
- Serum calcium < 8 mg/dI
- Fluid sequestration > 6 L
- Other scoring systems for acute pancreatitis are : Glasgow, apache II,

Correct Answer. a

(54). Risk factors for the development of basal cell carcinoma of the skin include all the following except

a. Fair skin with poor tanning capacity

b. Previous x-ray therapy for acne

c. Outside occupations

d. Dark skin

Solution. (d) Dark skin


Ref.: Read the text below
Sol :
- Basal cell carcinoma occurs most commonly on areas of the body that have been frequently exposed to the sun.
- UV radiation damages the skin, so previous x-ray therapy also causes some amount of skin damage and may predispose to future
development of this cancer.
- Occupations that are outside, such as farming, road work, house building, etc., also predispose the person to skin cancer.
- Dark skin is protective against UV radiation, and individuals having such skin have a lower incidence of skin cancers.

Correct Answer. d

Copyright © 2014 Delhi Academy of Medical Sciences, All Rights Reserved. 22/93
(55). 30 year old chain smoker presents with gangrene of lower limb. Biopsy shows neutrophilic involvement of vessel wall with nerve
involvement. What is the diagnosis?

a. Buerger’s disease

b. Raynaud’s disease

c. Atherosclerosis

d. A-V fistula

Solution. (a) Buerger’s disease


Ref.: Read the text below
Sol : Buerger’s Disease (Thromboangiitis Obliterans)
- Occlusive disease of small and medium sized arteries in distal upper and lower extremities (plantar, tibial, radial, ulnar)
- Usually occurs in male smokers
- Under the age of 30 years
- Histologically, there are inflammatory changes in the walls of arteries (panarteritis), veins and nerves.
- Internal elastic lamina is preserved and a cellular inflammatory thrombus develops in vascular lumen.
Clinical triad :
- Claudication of affected extremity
- Raynaud’s phenomenon
- Recurrent superficial thrombophlebitis (Phlebitis migrans)
Angiography :
- Smooth tapering segmental lesions in distal vessels (“cork-screw pattern”) are characteristic.
- Compression of a large AV fistula may cause reflex slowing of heart rate (Nicoladoni/Branham sign)
- Gold standard investigation for Aortic dissection is MRI
- Gold standard in v. for AV fistulas and Aneurysms is CT Angiography
- Most common cause of AV fistulas is surgically created > Traumatic.

Correct Answer. a

Copyright © 2014 Delhi Academy of Medical Sciences, All Rights Reserved. 23/93
(56). Instrument shown in the picture is?

a. Amputation saw

b. Humbey’s knife

c. Osteotome

d. Rib shear

Solution. (b) Humbey’s knife


Ref: Read the text below
Sol: This is a picture of Humbey’s knife. It is a manual dermatomes used to produce thin slices of skin from a donor area to be used for
skin grafting. It has an adjustable roller which controls the thickness of the graft.

Correct Answer. b

(57). Graft survival from 2nd to 4th day is based on?

a. Imbibition

b. Inosculation

c. Revascularisation

d. Neovascularisation

Solution. (b) Inosculation


Ref: Read the text below
Sol:

Correct Answer. b

Copyright © 2014 Delhi Academy of Medical Sciences, All Rights Reserved. 24/93
(58). Ideal time for cleft palate surgery is?

a. Within 3 months

b. 3 – 6 months

c. 4 months to 8 months

d. 6 months to 18 months

Solution. (d) 6 months to 18 months


Ref: Read the text below
Sol:
- Cleft palate: is done between 6 months to 18 months.
- If operated separately then soft palate is operated at 6m-12months soft palate at 6 months) and hard palate is operated at 12-18
months.
- If both are operated together than it is operated at around 1 year.

Correct Answer. d

(59). Central cleft lip is due to?

a. Failure of fusion between 2 central fronto nalsal processes

b. Failure of fusion between fronto-nasal process to maxillay process.

c. Failure of fusion between lateral fronto- nasal process tomaxilalry process

d. Failure of fusion beween 2 maxilary process

Solution. (a) Failure of fusion between 2 central fronto nalsal processes


Ref: Read the text below
Sol:

Correct Answer. a

Copyright © 2014 Delhi Academy of Medical Sciences, All Rights Reserved. 25/93
(60). Following IVP is showing which sign

a. Cobra head or Adder head sign of ureterocele

b. B/L VUR

c. Champagne glass sign of TCC ureter

d. Pine tree appearance of Neurogenic bladder

Solution. -NA-

Correct Answer. a

Copyright © 2014 Delhi Academy of Medical Sciences, All Rights Reserved. 26/93
(61). Most commonly performed surgery for PUJ obstruction is?

a. Anderson Hynes Pyeloplasty

b. Culp DeWeerd pyeloplasty

c. Scardino Prince Pyeloplasty

d. Foley’s Y-V plasty

Solution. (a) Anderson Hynes Pyeloplasty


Ref: Read the text below
Sol:

Correct Answer. a

Copyright © 2014 Delhi Academy of Medical Sciences, All Rights Reserved. 27/93
(62). A 45 years old female presents with pruritus and jaundice. She has raised ALP and GGT. Anti-mitochondrial antibody was positive. What
is the next line of investigation?

a. ERCP

b. MRCP

c. PET

d. Liver Biopsy

Solution. (b) MRCP


Ref.: Read the text below
Sol :
- MRCP is an excellent imaging technique for gall bladder and biliary system.
- It can demonstrate ductal obstruction, strictures or other intra ductal abnormalities.
- Contrast is not required.
- The patient in question is most probably a case of primary biliary cirrhosis.

Correct Answer. b

(63). Common bile duct injuries are most commonly seen in:

a. Radical gastrectomy

b. Penetrating injuries of abdomen

c. ERCP and sphincterotomy

d. Laparoscopic cholecystectomy operation

Solution. (d) Laparoscopic cholecystectomy operation


Ref.: CSDT 12th Ed. Pg. 597
Sol :
- Most common cause of bile duct stricture is common bile duct injury most commonly following laparoscopic surgery.
- Benign biliary injuries and stricture caused by surgical trauma in about 95% of cases.
- The no. of bile duct injuries has risen sharply in the past few years along with the shift from open to laparoscopic surgery.

Correct Answer. d

(64). Fibroblast in healing wound is derived from:

a. Local mesenchyma

b. Epithelium

c. Endothelium

d. Vascular fibrosis

Solution. (a) Local mesenchyma


Ref.: Sabiston Texbook of Surgery 17th Ed. Pg. 192
Sol :
- The primary function of fibroblasts is to synthesize collagen that they begin to produce during cellular phase of inflammation.
- The time required for undifferentiated mesenchymal cells to differentiate into highly specialized fibroblast accounts for delay between
injury and the appearance of collagen in a healing wound.

Correct Answer. a

Copyright © 2014 Delhi Academy of Medical Sciences, All Rights Reserved. 28/93
(65). What is most common type of carcinoma of the right colon?

a. Stenosing

b. Ulcerative

c. Tubular

d. Fungating

Solution. (b) Ulcerative


Ref.: Bailey and Love’s Short Practice of Surgery 4th Ed. Pg. 1179
Sol :
- Carcinoma of right side of colon is ulcerative type so it causes severe anemia and may present with malena, wile carcinoma of left side
of colon presents with obstruction do may present with altered bowel habit.

Correct Answer. b

(66). Which one of the following types of stomach cancers carries the best prognosis?

a. Superficial spreading type

b. Ulcerative type

c. Linitis plastic type

d. Polypoidal type

Solution. (a) Superficial spreading type


Ref.: Bailey and loves’ short practice of surgery -1051
Sol :
- Superficial spreading carcinoma (15%) also known as early gastric cancer is confined to mucosa and submucosa.
- Metastases present only in 30% of cases even if metastasis present prognosis after gastrectomy is much better than the other advanced
carcinoma.

Correct Answer. a

(67). Lymph node metastasis is common feature with the following variant of soft tissue sarcoma :

a. Fibrosarcoma

b. Angiosarcoma

c. Liposarcoma

d. Neurofibrosarcoma

Solution. (b) Angiosarcoma


Ref.: Read the text below
Sol :
- Angiosarcoma is a malignant neoplasm (cancer) of endothelial-type cells that line vessel walls. This may be in reference to blood
(hemangiosarcoma) or lymphatic vessels (lymphangiosarcoma).
- Its origin typically readily permits metastases to distant sites. Most tumors of visceral blood and lymphatic vessel walls are malignant.
Hemangiosarcomas and lymphangiosarcomas of the skin are not common.[
- Angiosarcoma of the liver, a rare fatal tumor, has been seen in workers intensively exposed to the gas vinyl chloride monomer (VCM)
for prolonged periods while working in polyvinyl chloride (PVC) polymerization plants. It has also been associated with individuals
exposed to arsenic-containing insecticides and Thorotrast

Correct Answer. b

Copyright © 2014 Delhi Academy of Medical Sciences, All Rights Reserved. 29/93
(68). Haagensen’s signs of inoperability of Carcinoma include :

a. Oedema of skin of breast or arm

b. Satellite tumor nodules in skin of breast

c. Proved supraclavicular or distant metastases

d. All of above

Solution. (d) All of above


Ref.: Read the text below
Sol : Haagensen’s Sign of inoperability
- Extensive edema of breast
- Supraclavicular metastasis
- Satellite nodules
- Inflammatory Carcinoma
- Distant metastasis
- Edema of Arm
- Parasternal tumor (indicating spread to internal mammary nodes)

Correct Answer. d

(69). A patient is brought to the emergency as a case of head injury, following a head on collision road traffic accident. His BP is 90/60 mmHg.
Tachycardia is present. Most likely diagnosis is :

a. EDH

b. SDH

c. Intracranial hemorrhage

d. Intra-abdominal bleed.

Solution. (d) Intra-abdominal bleed.


Ref.: Read the text below
Sol :
- Subdural hemorrhage is the most common intracranial complication following head trauma.
- But in this case patient is presented with severe hypotension of 90/60 mm Hg and has tachycardia which is not explained by extradural
or intracranial or subdural hemorrhage alone.
- So in the case the patient has an intra-abdominal source of bleed like a ruptured spleen which lead to severe hypotension and
tachycardia.

Correct Answer. d

(70). Which one of the following is the management of pancreatic abscess?

a. Needle aspiration

b. Gastrocystostomy

c. External drainage

d. Jejunocystostomy

Solution. (c) External drainage


Ref.: CSDT- 616
Sol :
- Collection of pus must be drained with percutaneous catheter as a first step to decrease toxicity or to obtain specimen for culture.
- In some cases, catheter drain proves to be sufficient but most often the infected retroperitoneal space contains necrotic debris that
needs surgical debridement because it cannot pass through catheter.
- Infect the two measures catheter drainage and debridement are complementary prophylactic.
- Broad – spectrum antibiotic should be given.

Correct Answer. c

Copyright © 2014 Delhi Academy of Medical Sciences, All Rights Reserved. 30/93
(71). Patients of increased risk for gastric carcinoma include the following except those:

a. Who have undergone gastric bypass for morbid obesity

b. Who have undergone gastric resection for gastric ulcer

c. With pernicious anaemia

d. With a high consumption of smoked fish

Solution. (a) Who have undergone gastric bypass for morbid obesity
Ref.: Harrison’s Principles of Internal Medicine- 525
Sol : Etiology of gastric carcinoma :
- Long – term ingestion of nitrates in dried smoked and salted food.
- Source of nitrate converting bacteria as a factor in the causation of gastric carcinoma.
- Exogenous source –
- Through the ingestion of partially decayed bacterially contaminated food (common in lower socioeconomic class)
- Helicobacter pylori infection.
- Endogenous factor favoring growth of nitrate converting bacteria in stomach:
- Decrease gastric acidity
- Prior gastric surgery (antrectomy) (15 to 20 years latency period)
- Atrophic gastritis and /or pernicious anemia
- Prolonged exposure to histamine H2 receptor blockers.

Correct Answer. a

Copyright © 2014 Delhi Academy of Medical Sciences, All Rights Reserved. 31/93
(72)
The most common cause of acute mesenteric ischaemia is
.

a. Arterial embolus

b. Arterial thrombus

c. Non occlusive mesenteric ischaemia

d. Venous thrombosis

Solution. (a) Arterial embolus


Ref: Read the text below
Sol:

Correct Answer. a

Copyright © 2014 Delhi Academy of Medical Sciences, All Rights Reserved. 32/93
(73). Site of needle puncture in tension pneumothorax

a. 2nd ICS midclavicular line

b. 5th ICS midaxillary line

c. 5th ICS in midclavicular line

d. 4th ICS

Solution. (a) 2nd ICS midclavicular line


Ref.: Read the text below
Sol :
- A chest tube (or intercostal drain) is the most definitive initial treatment of a pneumothorax.
- These are typically inserted in an area under the axilla (armpit) called the "safe triangle",2nd ICS midclavicular line where damage to
internal organs can be avoided; this is delineated by a horizontal line at the level of the nipple and two muscles of the chest wall
(latissimus dorsi and pectoralis major).
- Local anesthetic is applied. Two types of tubes may be used.
- In spontaneous pneumothorax, small-bore (smaller than 14 F, 4.7 mm diameter) tubes may be inserted by the Seldinger technique, and
larger tubes do not have an advantage.
- In traumatic pneumothorax, larger tubes (28 F, 9.3 mm) are used.

Correct Answer. a

(74). Which of the following conditions is most likely to follow a compression-type abdominal injury?

a. Renal vascular injury

b. Superior mesenteric thrombosis

c. Avulsion of the splenic pedicle

d. Diaphragmatic hernia

Solution. (d) Diaphragmatic hernia


Ref:Read the text below
Sol:
- In the rapiddeceleration injury associated with automobile crashes, the abdominal viscera tend to continue moving anteriorly after the
body wall has been stopped.
- These organs exert great stress upon the structures anchoring them to the retroperitoneum.
- Intestinal loops stretch and may tear their mesenteric attachments, injuring and thrombosing the superior mesenteric artery; kidneys
and spleen may similarly shear their vascular pedicles.
- In these injuries, however, ordinarily the intraabdominal pressure does not rise excessively and diaphragmatic hernia is not likely.
- Diaphragmatic hernia is primarily associated with compression-type abdominal or thoracic injuries that increase intraabdominal or
intrathoracic pressure sufficiently to tear the central portion of the diaphragm.

Correct Answer. d

(75). Nonoperative management of penetrating neck injuries has been advocated as an alternative to mandatory exploration in asymptomatic
patients. Which of the following findings would constitute a relative,rather than an absolute, indication for formal neck exploration?

a. Expanding hematoma

b. Dysphagia

c. Dysphonia

d. Pneumothorax

Solution. (d) Pneumothorax


Ref:Read the text below
Sol:
- Reports of a more than 50% incidence of negative explorations of the neck, iatrogenic complications, and serious injuries overlooked at
operation have caused a reassessment of the dictum that all penetrating neck wounds that violate the platysma must be explored.
- Pneumothorax would mandate a chest tube; the necessity for exploration would depend on clinical judgment and institutional policy.

Correct Answer. d

Copyright © 2014 Delhi Academy of Medical Sciences, All Rights Reserved. 33/93
(76). Following traumatic peripheral nerve transection, regrowth usually occurs at which of the following rates?

a. 0.1 mm per day

b. 1 mm per day

c. 5 mm per day

d. 1 cm per day

Solution. (b) 1 mm per day


Ref:Read the text below
Sol:
- Transection of a peripheral nerve results in hemorrhage and in retraction of the severed nerve ends.
- Almost immediately, degeneration of the axon distal to the injury begins.
- Degeneration also occurs in the proximal fragment back to the fist node of Ranvier.
- Phagocytosis of the degenerated axonal fragments leaves a neurilemmal sheath with empty cylindrical spaces where the axons were.
- Several days following the injury, axons from the proximal fragment begin to regrow.
- If they make contact with the distal neurilemmal sheath, regrowth occurs at about the rate of 1 mm per day.

Correct Answer. b

(77). A 36-year-old man sustains a gunshot wound to the left buttock. He is hemodynamically stable.There is no exit wound, and an X ray of
the abdomen shows the bullet to be located in the right lower quadrant. Correct management of a suspected rectal injury would include

a. Barium studies of the colon and rectum

b. Barium studies of the bullet track

c. Endoscopy of the bullet track

d. Sigmoidoscopy in the emergency room

Solution. (d) Sigmoidoscopy in the emergency room


Ref:Read the text below
Sol:
- Penetrating injury to the intraperitoneal or extraperitoneal rectum should be diagnosed by immediate sigmoidoscopy.
- Contrast studies of the rectum, when sigmoidoscopy is inconclusive, should use a watersoluble radiopaque medium such as
Gastrografin.
- The use of barium is contraindicated because its spillage in the peritoneal cavity mixed with feces would increase the likelihood of
subsequent intraabdominal abscesses.
- Instrumentation of the bullet track is also contraindicated because of the risk of injury to adjacent structures (e.g., bladder, ureters,iliac
vessels). Angiography is not a sensitive method for demonstrating injury of the intestinal wall.

Correct Answer. d

(78). The diagnostic tests performed reveal extravasation of contrast into the renal parenchyma.Treatment should consist of

a. Exploration and suture of the laceration

b. Exploration and wedge resection of the left kidney

c. Nephrostomy

d. Antibiotics and serial monitoring of blood count and vital signs

Solution. (d) Antibiotics and serial monitoring of blood count and vital signs
Ref:Read the text below
Sol:
- Seventy to eighty percent of patients with blunt renal trauma are successfully treated nonsurgically.

Correct Answer. d

Copyright © 2014 Delhi Academy of Medical Sciences, All Rights Reserved. 34/93
(79). True statements concerning penetrating pancreatic trauma include

a. Most injuries do not involve adjacent organs

b. Management of a ductal injury in the head of the pancreas is pancreaticoduodenectomy

c. Small peripancreatic hematomas need not be explored to search for pancreatic injury

d. The major cause of death is exsanguinations from associated vascular injuries

Solution. (d) The major cause of death is exsanguinations from associated vascular injuries
Ref:Read the text below
Sol:
- The majority of penetrating pancreatic injuries can be managed with simple drainage.
- Injury to the major pancreatic duct to the left of the mesenteric vessels is effectively treated with a distal pancreatectomy.
- The high morbidity and mortality of a pancreaticoduodenectomy for trauma limit its use to extensive blunt injuries to both pancreatic
head and duodenum.
- For ductal injury in the region of the head of the pancreas, a Roux-en-Y limb of jejunum should be brought up and used to drain the
transected duct.
- The proximity of the pancreas to many other major structures makes combined injuries frequent (90%).
- Complications of pancreatic injury include fistula, pseudocyst, and abscess, but the cause of death in patients with pancreatic injury is
most frequently exsanguination from associated injury to major vascular structures such as the splenic vessels, mesenteric vessels, aorta,
or inferior vena cava.
- Finally, however small, all peripancreatic hematomas should be explored to search for pancreatic injury. Simple drainage is usually
adequate treatment in such cases, but failure to recognize a pancreatic injury can have catastrophic sequelae.

Correct Answer. d

(80). Which statement concerning radiation-induced thyroid cancer is true?

a. It usually follows high-dose radiation to the head and neck

b. A patient with a history of radiation is safe if no cancer has been found 20 years after exposure

c. Most radiation-induced thyroid cancers are follicular

d. The treatment of choice is a neartotal (or total) thyroidectomy

Solution. (d) The treatment of choice is a neartotal (or total) thyroidectomy


Ref:Read the text below
Sol:
- Radiation-induced thyroid cancer usually follows low-dose external radiation.
- Most cancers occur after exposure to 1500 rads or less to the neck, but an increase in thyroid cancer has been noted after as little as 6
rads.
- Salivary gland tumors and possibly parathyroid adenomas are also associated with radiation.
- The latent period for these tumors is 30 years or longer. Of all patients who have low-dose radiation,about 9% have been found to have
thyroid cancer, usually of the papillary type.
- Treatment consists of a near-total thyroidectomy because there is a high incidence of bilaterality and because there is a greater
incidence of complications if a second operation is necessary.

Correct Answer. d

Copyright © 2014 Delhi Academy of Medical Sciences, All Rights Reserved. 35/93
(81). As an incidental finding during an upper abdominal CT scan, a 3-cm mass in the adrenal gland is noted. The appropriate next step in
analysis and management of this finding would be

a. Observation

b. CT-guided needle biopsy

c. Excision of the mass

d. Measurement of urine catecholamine excretion

Solution. (a) Observation


Ref:Read the text below
Sol:
- With the increasing use of CT and MRI scans for other purposes, small “incidentalomas” of the adrenal gland are becoming a frequent
finding.
- In the absence of any clinical signs or symptoms of endocrine dysfunction, most experts now recommend observation and a search for
evidence of endocrine dysfunction for lesions less than 5 cm in diameter.

Correct Answer. a

(82). A 67-year-old smoker undergoes a chest CT scan to evaluate a 1-cm lung nodule found on chest x-ray. The CT scan demonstrates
mediastinal adenopathy. The next step in the workup should be:

a. Magnetic resonance imaging (MRI) scan of the chest and abdomen

b. Chemotherapy consultation

c. Radiotherapy consultation

d. Bronchoscopy and mediastinoscopy

Solution. (d) Bronchoscopy and mediastinoscopy


Ref:Read the text below
Sol
- Evaluation of a lung tumor found on radiologic imaging should include thorough assessment of the primary tumor site as well as a
search for local and distant metastatic disease.
- Assuming a negative metastatic workup, the finding of mediastinal adenopathy in association with a lung tumor requires biopsy for
diagnosis.
- This is often best approached via a bronchoscopy and mediastinoscopy, with biopsy of the mediastinal lymph nodes to evaluate the
stage of the disease before further treatment.

Correct Answer. d

(83). Protein metabolism after trauma is characterized by the following except –

a. Increased liver gluconeogenesis

b. Inhibition of skeletal muscle breakdown by interleukin 1 and tumour necrosis factor

c. Increased urinary nitrogen loss

d. Hepatic synthesis of acute phase reactants.

Solution. (b) Inhibition of skeletal muscle breakdown by interleukin 1 and tumour necrosis factor
Ref.: Read the text below
Sol : PROTEIN METABOLISM AFTER TRAUMA
a. Increased liver gluconeogenesis
b. Increased urinary nitrogen loss
c. Hepatic synthesis of acute phase reactants.

Correct Answer. b

Copyright © 2014 Delhi Academy of Medical Sciences, All Rights Reserved. 36/93
(84). Most sensitive screening procedure for CA Prostate

a. DRE

b. PSA

c. DRE + PSA

d. Endorectal coil MRI with T1W & T2W images

Solution. (c) DRE + PSA


Ref:Read the text below
Sol:
- The most common prostate cancer screening modalities are DRE and serum prostate-specific antigen (PSA) assay.
- Newer serum tests, such as measurement of bound to free serum PSA, have yet to be fully evaluated.
- This disease is prone to lead-time bias, length bias, and overdiagnosis, and substantial debate rages among experts as to whether it is
effective.
- Prostate cancer screening clearly detects many asymptomatic cancers, but the ability to distinguish tumors that are lethal but still
curable from those that pose little or no threat to health is limited.
- Men over age 50 have a high prevalence of indolent, clinically insignificant prostate cancers.

Correct Answer. c

(85). Orchiopexy for undescended testes is done ideally at what age?

a. Neonate

b. 1-2 yrs

c. 5 yrs

d. Puberty

Solution. (a) Neonate


Ref: Nelson Textbook of Pediatrics, 18th ed
Sol: UNDESCENDED TESTES
- Failure to find the testis in the scrotum indicates that the testis is undescended, absent, or retractile.
EPIDEMIOLOGY.
- An undescended (cryptorchid) testis is the most common disorder of sexual differentiation in boys.
- At birth, approximately 4.5% of boys have an undescended testis.
- Because testicular descent occurs late in gestation, 30% of premature male infants have an undescended testis; the incidence is 3.4% at
term.
- The majority of undescended testes descend spontaneously during the first 3 mo of life, and by 6 mo the incidence decreases to 0.8%.
- If the testis has not descended by 4 mo, it will remain undescended. Cryptorchidism is bilateral in 10% of cases.
CLINICAL MANIFESTATIONS.
- Undescended testes can be classified as abdominal (nonpalpable), peeping (abdominal but can be pushed into the upper part of the
inguinal canal), inguinal, gliding (can be pushed into the scrotum but retracts immediately to the pubic tubercle), and ectopic (superficial
inguinal pouch or, rarely, perineal).
- Most undescended testes are in or just distal to the inguinal canal.
- Approximately 10% of cryptorchid boys have a nonpalpable testis; in these boys the testis is present and abdominal or inguinal in 50%,
and it is absent secondary to perinatal testicular torsion in 50%.
- In a newborn phenotypic male with bilateral nonpalpable testes, one should consider the possibility that the child could be a virilized
female with congenital adrenal hyperplasia
- In a boy with an undescended testis and midpenile or proximal hypospadias, an intersex condition should be considered.The
consequences of cryptorchidism include infertility, testicular malignancy, associated hernia, torsion of the cryptor-chid testis, and the
possible psychologic effects of an empty scrotum.
- The undescended testis is normal at birth histologically, but pathologic changes can be demonstrated by 6–12 mo.
- Delayed germ cell maturation, reduction in germ cell number, hyalinization of the seminiferous tubules, and reduced Leydig cell
number are typical; these changes are progressive over time if the testis remains undescended.
- Similar, although less severe, changes are found in the contralateral descended testis after 4–7 yr.
- After treatment for a unilateral undescended testis, 85% of patients are fertile, which is slightly less than the 90% rate of fertility in an
unselected population of male adults. In contrast, following bilateral orchiopexy, only 50–65% of patients are fertile.
TREATMENT.
- The congenital undescended testis should be treated surgically no later than 9–15 mo.
- With anesthesia by a pediatric anesthesiologist, surgical correction at 6 mo is appropriate, because spontaneous descent of the testis
will not occur after 4 mo of age.
- Most testes can be brought down to the scrotum with an orchiopexy, which involves an inguinal incision, mobilization of the testis and
spermatic cord, and correction of the indirect inguinal hernia.
- The procedure is typically performed on an outpatient basis and has a success rate of 98%.
- In some boys with a testis that is close to the scrotum, a Bianchi or prescrotal orchiopexy can be performed. In this procedure, the
entire operation is performed through an incision along the edge of the scrotum.

Correct Answer. a

Copyright © 2014 Delhi Academy of Medical Sciences, All Rights Reserved. 37/93
(86). Polyp not associated with risk of malignancy

a. Juvenile polyp

b. FAP

c. Peutz Jeghers syndrome

d. Juvenile polyposis syndrome

Solution. (a) Juvenile polyp


Ref: Schwartz's Principles of Surgery, Ninth Edition
Sol:
Hamartomatous Polyps (Juvenile Polyps)
- In contrast to adenomatous polyps, hamartomatous polyps (juvenile polyps) usually are not premalignant.
- These lesions are the characteristic polyps of childhood but may occur at any age.
- Bleeding is a common symptom and intussusception and/or obstruction may occur.
- Because the gross appearance of these polyps is identical to adenomatous polyps, these lesions should also be treated by polypectomy.
In contrast to adenomatous polyposis syndromes, these conditions are often associated with mutation in PTEN.
- Familial juvenile polyposis is an autosomal dominant disorder in which patients develop hundreds of polyps in the colon and rectum.
- Unlike solitary juvenile polyps, these lesions may degenerate into adenomas and, eventually, carcinoma.
- Annual screening should begin between the ages of 10 and 12 years.
- Treatment is surgical and depends, in part, upon the degree of rectal involvement.
- If the rectum is relatively spared, a total abdominal colectomy with ileorectal anastomosis may be performed with subsequent close
surveillance of the retained rectum.
- If the rectum is carpeted with polyps, total proctocolectomy is the more appropriate operation. These patients are candidates for ileal
pouch–anal reconstruction to avoid a permanent stoma.
- Peutz-Jeghers syndrome is characterized by polyposis of the small intestine and, to a lesser extent, polyposis of the colon and rectum.
- Characteristic melanin spots often are noted on the buccal mucosa and lips of these patients.
- The polyps of Peutz-Jeghers syndrome generally are considered to be hamartomas and are not thought to be at significant risk for
malignant degeneration.
- However, carcinoma may occasionally develop.
- Because the entire length of the GI tract may be affected, surgery is reserved for symptoms such as obstruction or bleeding or for
patients in whom polyps develop adenomatous features.
- Screening consists of a baseline colonoscopy and upper endoscopy at age 20 years, followed by annual flexible sigmoidoscopy
thereafter.
- Cronkite-Canada syndrome is a disorder in which patients develop GI polyposis in association with alopecia, cutaneous pigmentation,
and atrophy of the fingernails and toenails. Diarrhea is a prominent symptom, and vomiting, malabsorption, and protein-losing
enteropathy may occur. Most patients die of this disease despite maximal medical therapy, and surgery is reserved for complications of
polyposis, such as obstruction.
- Cowden syndrome is an autosomal dominant disorder with hamartomas of all three embryonal cell layers. Facial trichilemmomas,
breast cancer, thyroid disease, and GI polyps are typical of the syndrome. Patients should be screened for cancers. Treatment is
otherwise based upon symptoms.

Correct Answer. a

(87). Renal calculi associated with proteus infection

a. Uric acid

b. Triple phosphate

c. Calcium oxalate

d. Xanthine

Solution. (b) Triple phosphate


Ref:Read the text below
Sol: Magnesium-Ammonium-Phosphate Stones
- These stones are usually secondary to urinary tract infection due to bacteria that produce urease (primarily proteus species).
- Eradication of the infection will prevent further stone formation but is impossible when stones are present.
- After all calculi have been removed, prevention of stone growth is best accomplished by urinary acidification, long-term use of
antibiotics, and, perhaps, use of acetohydroxamic acid, a urease inhibitor that maintains an acid urinary pH and may potentiate antibiotic
action.

Correct Answer. b

Copyright © 2014 Delhi Academy of Medical Sciences, All Rights Reserved. 38/93
(88). The diagnostic imaging modality of choice for retroperitoneal infections is

a. Barium swallow

b. MRI

c. CT scan

d. Endoscopy

Solution. (c) CT scan


Sol: RETROPERITONEAL INFECTIONS
- The source of retroperitoneal infections is usually an organ contained within
or abutting the retroperitoneum.
- Retrocecal appendicitis, perforated duodenal ulcers, pancreatitis, or diverticulitis may all lead to retroperitoneal infection with or
without abscess formation.
Clinical features
- Patients with a retroperitoneal abscess usually present with pain, fever, and malaise.
- Tachypnea and tachycardia.
- Laboratory evaluation usually reveals a leukocytosis.
Diagnosis
- The diagnostic imaging modality of choice is abdominal CT.
Management
- Unilocular abscesses are usually drained percutaneously under CT guidance.
-Multilocular collections may require operative intervention for adequate drainage.

Correct Answer. c

(89). The VIPoma syndrome also is called the WDHA syndrome because of the presence of ;

a. Watery diarrhea, hypokalemia, and achlorhydria

b. Watery diarrhea, hyperchloremia and achlorhydria

c. Watery diarrhea, hyperkalemia, and achlorhydria

d. Watery diarrhea, hypochloremia, and achlorhydria

Solution. (a) Watery diarrhea, hypokalemia, and achlorhydria


Sol VIPoma
- Syndrome associated with a pancreatic neoplasm secreting vasoactive intestinal polypeptide (VIP).
- The VIPoma syndrome also is called the WDHA syndrome because of the presence of watery diarrhea, hypokalemia, and achlorhydria.
- Serum VIP levels must be measured on multiple occasions because the excess secretion of VIP is episodic, and single measurements
might be normal and misleading.
- A CT scan localizes most VIPomas, although as with all islet cell tumors, EUS is the most sensitive imaging method.
- VIPomas are more commonly located in the distal pancreas and most have spread outside the pancreas.
- Palliative debulking operations can sometimes improve symptoms for a period along with somatostatin analogs.
- Hepatic artery embolization also has been reported as a potentially beneficial treatment

Correct Answer. a

Copyright © 2014 Delhi Academy of Medical Sciences, All Rights Reserved. 39/93
(90). True statements regarding squamous cell carcinoma of the lip include

a. The lesion often arises in areas of persistent hyperkeratosis

b. More than 90% of cases occur on the upper lip

c. The lesion constitutes 30% of all cancers of the oral cavity

d. Radiotherapy is considered inappropriate treatment for these lesions

Solution. (a) The lesion often arises in areas of persistent hyperkeratosis


Ref:Read the text below
Sol:
- Squamous cell carcinoma of the lip is the most common malignant tumor of the lip and constitutes15% of all malignancies of the oral
cavity.
- Basal cell carcinomas do occur on the lip, but much less frequently. There is a strong association between squamous cell tumors of the
lip and sun exposure.
- Because of its greater sun exposure, the lower lip is the site of more than 90% of such lesions.
- Persistent hyperkeratosis precedes 35–40% of these lesions.
- These lip tumors are very responsive to radiotherapy, which works well for small to medium-sized lesions. Large lesions treated with
radiotherapy usually require surgical reconstruction.
- Radiotherapy should not be used in patients who will have ongoing sun exposure to the area because radiation therapy sensitizes the
tissues to solar trauma.

Correct Answer. a

(91). The usual incision given for Zenker's diverticulum is

a. Left Cervical

b. Right Cervical

c. Suprahyoid

d. Midline

Solution. (a) Left Cervical


Ref:Read the text below
Sol:
- The pharyngoesophageal diverticulum (Zenker’s diverticulum) is the most common esophageal diverticulum. Zenker’s diverticulum
usually presents in patients older than 60 years.
- Treatment can be done endoscopically or surgically. A commonly used surgical approach is cervical esophagomyotomy and resection of
the diverticulum performed through an oblique left cervical incision that parallels the anterior border of the sternocleidomastoid muscle
or a transverse cervical incision centered over the cricoid cartilage.

Correct Answer. a

Copyright © 2014 Delhi Academy of Medical Sciences, All Rights Reserved. 40/93
(92). Which of the following is true with regard to wound contraction?

a. is the primary process affecting closure of a sutured or stapled surgical wound

b. Bacterial colonization significantly slows the process of contraction

c. It may account for a maximum of 50% decrease in the size of a wound

d. It is based on specialized fibroblasts that contain actin myofilaments

Solution. (d) It is based on specialized fibroblasts that contain actin myofilaments


Ref:Read the text below
Sol:
- While epithelialization is responsible for the healing of a closed incision, wound contraction is the primary method of closure in open
wounds.
- During this process, the skin surrounding the wound is pulled over the wound surface and may account for up to a 90% reduction in the
size of an open wound.
- In areas of greater adherence of skin to underlying tissue, the ability of contraction to close the wound is hindered due to the decreased
mobility of the skin.
- Therefore, in areas of tight skin adherence such as the leg, contraction may only account for 30–40% reduction in wound size.
- Fibroblasts in the open wound, which predominate during the proliferative phase, contain increasing numbers of actin microfilaments,
thereby becoming myofibroblasts.

Correct Answer. d

(93). Peritoneo-Venous shunt is contraindicated in

a. Uncorrectable coagulopathy

b. Spontaneous bacterial peritonitis

c. Hepatic encephalopathy

d. All

Solution. (d) All


Ref: Maingot 1640
Sol:
- These shunts were used for intractable ascites in the past but due to high rate of complications there is no role for these shunts in
cirrhotic ascites.
- Bleeding occurs due to DIC, with prolonged PT, aPTT.
- DIC is caused by an activator in ascitc fluid. Bacterial Peritonitis is staph induced.
- Failure of shunts is 50% in 1st year

Correct Answer. d

Copyright © 2014 Delhi Academy of Medical Sciences, All Rights Reserved. 41/93
(94). “Butterfly shaped hematoma”is a characteristic feature of: -

a. Extraperitoneal bladder rupture

b. Intraperitoneal bladder rupture

c. Anterior urethral rupture

d. Posterior urethral rupture

Solution. (c) Anterior urethral rupture


Reference – Sch-15345-46
Sol

Correct Answer. c

Copyright © 2014 Delhi Academy of Medical Sciences, All Rights Reserved. 42/93
(95). A 28-year-old man discovers a mass in his neck while buttoning his shirt collar. Physical examination reveals a 2 cm mass in one thyroid
lobe, which is "cold" on scintiscan. Aspiration of the nodule demonstrates small "solid balls" of neoplastic follicular cells. Careful
examination of these tissue balls reveals that they contain microscopic blood vessels and fibrous stroma in their centers. Which of the
following is the most likely diagnosis?

a. Follicular carcinoma

b. Hashimoto's disease

c. Medullary carcinoma

d. Papillary carcinoma

Solution. (d) Papillary carcinoma


Reference – Read the text below
Sol:
- The distinctive cell balls described are broken-off papillary clusters, and are considered pathognomic for papillary carcinoma of the
thyroid.
- This is the most common form of thyroid carcinoma.
- It tends to present in the 3rd to 5th decade and shows a modest female predominance. Despite its propensity for local lymphatic
intrusion (which may cause multifocality of tumor in the thyroid or cervical lymph node metastases), the tumor generally has an excellent
prognosis with 90% 20-year survival.

Correct Answer. d

(96). Duodenoduodenostomy is surgical treatment of choice for all of following except?

a. Duodenal stenosis

b. Duodenal atresia

c. Annular pancreas

d. Duodenal web

Solution. (d) Duodenal web


Reference – Read the text below
Sol:
- The surgical treatment of choice for duodenal obstruction due to duodenal stenosis or atresia or annular pancreas is a
duodenoduodenostomy. This procedure can be most easily performed using a proximal transverse-to-distal longitudinal (diamond-shaped)
anastomosis.
-Treatment of duodenal web includes vertical duodenotomy, excision of the web, oversewing of the mucosa, and closing the duodenotomy
horizontally.
Ans 97: (b) Urinary tract infection Reference – Read the text below Sol:
- Hypospadias, which is congenital displacement of the urethral opening onto the ventral surface (underside) of the penis, is due to
malformation of the urethral groove and canal.
- Hypospadias frequently accompanies other genitourinary anomalies, especially cryptorchidism.
- Isolated hypospadias is repaired because the abnormal opening is often constricted, leading to urinary retention and ascending urinary
tract infections.

Correct Answer. d

Copyright © 2014 Delhi Academy of Medical Sciences, All Rights Reserved. 43/93
(97). A newborn male child is noted to have hypospadias. A complete evaluation determines that the child has no other genitourinary
anomalies. Nonetheless, hypospadias repair will be performed to prevent which of the following possible sequelae?

a. Hydrocele

b. Urinary tract infection

c. Phimosis

d. Urachal cysts

Solution. (b) Urinary tract infection


Reference – Read the text below
Sol:
- Hypospadias, which is congenital displacement of the urethral opening onto the ventral surface (underside) of the penis, is due to
malformation of the urethral groove and canal.
- Hypospadias frequently accompanies other genitourinary anomalies, especially cryptorchidism.
- Isolated hypospadias is repaired because the abnormal opening is often constricted, leading to urinary retention and ascending urinary
tract infections.

Correct Answer. b

(98). A surgeon wishes to perform a splenectomy on a patient who has been in an automobile accident. Before removing the spleen, the
splenic artery and splenic vein are ligated. Within which of the following peritoneal structures are the splenic artery and vein found?

a. Gastrocolic ligament

b. Gastrosplenic ligament

c. Splenorenal ligament

d. Lesser omentum

Solution. (c) Splenorenal ligament


Reference – Read the text below
Sol:
- The splenorenal ligament is the portion of the dorsal mesentery between the posterior abdominal wall and the spleen.
- This mesentery transmits the splenic artery and vein from their retroperitoneal position in the proximal portion of their course to the
peritoneal spleen.

Correct Answer. c

(99). In which of the following sites do myxopapillary ependymomas most frequently occur?

a. Cerebellum

b. Conus medullaris

c. 4th ventricle

d. Lateral ventricles

Solution. (b) Conus medullaris


Reference – Read the text below
Sol:
- Myxopapillary ependymoma is a variant of ependymoma, a tumor arising from ependymal cells.
- Histologically, myxopapillary ependymoma contains a myxoid (mucus-rich) intercellular matrix, in which spindly neoplastic ependymal
cells are arranged in a fascicular and papillary pattern (hence its designation).
- It is a benign tumor that almost always occurs in the distal segment of the spinal cord, ie, the conus medullaris.

Correct Answer. b

Copyright © 2014 Delhi Academy of Medical Sciences, All Rights Reserved. 44/93
(100). A 35-year-old woman notices a change in the appearance of a mole on her neck. Physical examination reveals that the lesion is an
irregular, nodular, superficial mass with a variegated appearance. Biopsy demonstrates a primary malignant tumor. Which of the
following factors is most predictive of the patient's long term prognosis?

a. Depth of the lesion

b. Circumference of the lesion

c. Darkness or degree of variation in color

d. None

Solution. (a) Depth of the lesion


Reference – Read the text below
Sol:
- The lesion is a malignant melanoma. Melanomas can develop either de novo or in an existing mole. Sunlight exposure is a significant
risk factor and fair-skinned persons are at increased risk of developing melanoma.
- The most significant factor for long term prognosis is the depth of the lesion, since the superficial dermis lies about 1 mm under the
skin surface, and penetration to this depth is associated with a much higher incidence of metastasis than is seen with a more superficial
location.
- The circumference of the lesion is much less important than depth, since one form of melanoma (superficial spreading) can still have
good prognosis despite large size, if it has not extended to the depth of the superficial dermal lymphatic bed.
- The darkness or degree of variation in color do not have prognostic significance once melanoma is diagnosed.

Correct Answer. a

(101). A patient has a painful ulcer on the tip of his tongue. Which of the following cranial nerves carries the pain sensation he experiences?

a. V2

b. V3

c. VII

d. IX

Solution. (b) V3
Reference – Read the text below
Sol:
- The innervation of the tongue is complex. The mandibular division of the trigeminal nerve (V3) carries general somatic sensation from
the anterior two-thirds of the tongue.
- The maxillary division (V2) carries somatic sensation from the palate, upper gums, and upper lip.
- The facial nerve (VII) carries taste from the anterior two-thirds of the tongue.
- The glossopharyngeal nerve carries sensation and taste from the posterior one-third of the tongue.

Correct Answer. b

(102). 54-year-old male with acute lymphocytic leukemia develops a blast crisis. He is treated with intensive systemic chemotherapy. Following
treatment, the patient will be at increased risk for the development of

a. Bile Pigment Gallstones

b. Cholesterol Gallstones

c. Cystine Kidney Stones

d. Uric Acid Kidney Stones

Solution. (d) Uric Acid Kidney Stones


Reference – Read the text below
Sol:
- Uric acid kidney stones in patients with leukemia are secondary to increased production of uric acid from purine breakdown during
periods of active cell proliferation, especially following treatment.
- Vigorous hydration and diuresis are generally instituted after the diagnosis of acute leukemia is made.
- Uric acid kidney stones are also associated with inborn errors of purine metabolism, such as gout.

Correct Answer. d

Copyright © 2014 Delhi Academy of Medical Sciences, All Rights Reserved. 45/93
(103). A man with blunt abdominal trauma with h/o pelvic fracture has presented ER. He has passed only few drops of blood per meatus and no
urine in the past 8 hrs. His bladder is palpable per abdomen. Which of the following is correct?

a. Anuria due to hemorrhagic shock

b. Urethral injury

c. Ureteral rupture leading to extravasation of urine in retroperitoneum

d. Bladder rupture

Solution. (b) Urethral injury


Ref:Read the text below
Sol: Injuries to the Ureter
Essentials of Diagnosis
- Anuria or oliguria; prolonged ileus or flank pain following pelvic operation.
- Onset of urinary drainage through wound or vagina.
- Demonstration of urinary extravasation or ureteral obstruction by urography.
Most ureteral injuries are iatrogenic in the course of pelvic surgery.
- Ureteral injury may occur during transurethral bladder or prostate resection or ureteral manipulation for stone or tumor.
- Ureteral injury is rarely a consequence of penetrating trauma.
- Unintentional ureteral ligation during operation on adjacent organs may be asymptomatic, though hydronephrosis and loss of renal
function will result.
- Ureteral division leads to extravasation and ureterocutaneous fistula.
Symptoms
- If the ureteral injury is not recognized at surgery, the patient may complain of flank and lower abdominal pain on the injured side. Ileus
and pyelonephritis may develop. Later, urine may drain through the wound (or through the vagina following transvaginal surgery).
- Wound drainage may be evaluated by comparing creatinine levels found in the drainage fluid with serum levels: urine exhibits very high
creatinine levels when compared with serum.
- Intravenous administration of 5 mL of indigo carmine will cause the urine to appear blue-green; therefore, drainage from a
ureterocutaneous fistula becomes blue, while serous drainage remains yellow. Anuria following pelvic surgery not responding to
intravenous fluids means bilateral ureteral ligation until proved otherwise.
- Peritoneal signs may occur if urine leaks into the peritoneal cavity

Correct Answer. b

(104). Sister Mary Joseph nodule associated with

a. Pancreatic ca

b. Gastric ca

c. Colonic ca

d. Ovarian ca

Solution. (b) Gastric ca


Ref:Read the text below
Sol:
- Gastric carcinomas spread by direct extension through the gastric wall to the perigastric tissues, occasionally adhering to adjacent
organs such as the pancreas, colon, or liver. The disease also spreads via lymphatics or by seeding of peritoneal surfaces.
- Metastases to intraabdominal and supraclavicular lymph nodes occur frequently, as do metastatic nodules to the ovary (Krukenberg's
tumor), periumbilical region ("Sister Mary Joseph node"), or peritoneal cul-de-sac (Blumer's shelf palpable on rectal or vaginal
examination); malignant ascites may also develop.
- The liver is the most common site for hematogenous spread of tumor.

Correct Answer. b

Copyright © 2014 Delhi Academy of Medical Sciences, All Rights Reserved. 46/93
(105). True about pneumococcal peritonitis is :

a. Commoner in girls

b. Prevalence has been increasing is recent years due to usage of tampoons

c. Clinically is much more aggressive than perforated appendicitis

d. Laparotomy is usually not necessary

Solution. (a) Commoner in girls


Ref: (Ref. Bailey & Love-999)
Sol:
- Pneumococal peritonitis may be primary or secondary.
- Primary form is commoner. It is common in young under nourished girls between ages of 3-6 years.
- The organisms gain access to peritoneal cavity from the fallopian tubes.
- Early surgery by laparotomy or laparoscopy is usually required.

Correct Answer. a

(106). All of the following are true about distributive shock, except :

a. High cardiac output

b. High vascular resistance

c. Low CVP

d. High Base deficit

Solution. (b) High vascular resistance


Ref: (Ref. Bailey & Love- 14
Sol:

Correct Answer. b

Copyright © 2014 Delhi Academy of Medical Sciences, All Rights Reserved. 47/93
(107). ‘Ebb & flow phase’ theory was given by :

a. Sir David Cutherbuton

b. Claude Bernard

c. Walter Cannon

d. John Hunter

Solution. (a) Sir David Cutherbuton


Ref:Bailey & Love’s, -- 5-6)
Sol:
- In 1930, Sir David Cuthbertson divided the metabolic response to injury in humans into ‘ebb’ and ‘flow’ phases. The ebb phase begins at
the time of injury and lasts for approximately 24-48 hours. It may be attenuated by proper resuscitation, but not completely abolished.
- The ebb phase is characterized by hypovolaemia, decreased basal metabolic rate, reduced cardiac output, hypothermia and lactic
acidosis.The predominant hormones regulating the ebb phase are catecholamines, cortisol system.
- The magnitude of this neuroendocrine response depends on the degree of blood loss and the stimulation of somatic afferent nerves at
the site of injury. The main physiological role of the ebb phase is to conserve both circulating volume and energy stores for recovery and
repair.
- Following resuscitation, the ebb phase evolves into a hypermetabolic flow phase, which corresponds to the SIRS. This phase involves
the mobilization of body energy stores for recovery and repair, and the subsequent replacement of lost or damaged tissue.
- It is characterized by tissue oedema (from vasodilatation and increased capillary leakage), increased basal metabolic rate
(hypermetabolism), increased cardiac output, raised body temperature. leucocytosis, increased oxygen consumption and increased
gluconeogenesis.
- The flow phase may be subdivided into an initial catabolic phase, lasting approximately 3-10 days, followed by an anabolic phase, which
may last for weeks if extensive recovery and repair are required following serious injury.
- During the catabolic phase, the increased production of counter-regulatory hormones (including catecholamines, cortisol, insulin and
glucagons) and inflammatory cytokines (e.g. IL-1, IL-6 and TNF) results in significant fat and protein mobilization, leading to significant
weight loss and increased urinary nitrogen excretion.

Correct Answer. a

Copyright © 2014 Delhi Academy of Medical Sciences, All Rights Reserved. 48/93
(108). PIRO classification scheme is used for :

a. Burns

b. Pre-Anaesthetic evaluation

c. Sepsis

d. Hypothermia

Solution. (c) Sepsis


Ref: Schwartz’s, - 112-113)
Sol:
- Clinicians dedicated to improving the treatment of sepsis have recently developed a new classification scheme for this entity
- This scheme has borrowed from the tumor-node-metastasis (TNM) staging scheme developed for oncology.
- The impetus for development of this scheme was related to the heterogeneity of the patient population developing sepsis, an example of which
would include two patients, both in the intensive care unit (ICU), who develop criteria consistent with septic shock.
- While both have infection and sepsis-associated hypotension, one might expect a different outcome in a young, healthy patient who develops
urosepsis than in an elderly, sive fungal infection. The PIRO Staging System stratifies patients based on their predisposing conditions (P), the
nature and extent (R), and the degree of concomitant organ dysfunction (O).
- Clinical trials evaluating the utility of this scoring system for prognostication and for examination of the impact of adjuvant therapies are
underway.

Correct Answer. c

Copyright © 2014 Delhi Academy of Medical Sciences, All Rights Reserved. 49/93
(109). Peutz Jeghers polyps present in

a. Rectum

b. Colon

c. Esophagus

d. Jejunum

Solution. (d) Jejunum


Ref: Read the text below
Sol:

Correct Answer. d

Copyright © 2014 Delhi Academy of Medical Sciences, All Rights Reserved. 50/93
(110). Bier’s block used in treatment of burns are :

a. Hydro fluoric acid

b. Hydro chloric acid

c. Sulphuric acid

d. Concentrated Nitric acid

Solution. (a) Hydro fluoric acid


Ref:Bailey and Love-392)
Sol:
- One acid that is a common cause of acid burns is hydrofluoric acid. Burns affecting the fingers and caused by dilute acid are relatively
common.
- The initial management is with calcium gluconate gel topically; however, severe burns or burns to large areas of the hand can be
subsequently treated with Bier’s blocks containing calcium gluconate 10% gel.
- If the patient has been burnt with a concentration greater than 50% the threat of hypocalcaemia and subsequent arrhythmias then
becomes high, and this is an indications for acute early excision.
- It is best not to do this at a delayed stage.

Correct Answer. a

(111). Single most important risk factor for chronic rejection is :

a. HLA match

b. Old is chamic time

c. Previous episodes of acute rejection

d. CMV injection

Solution. (c) Previous episodes of acute rejection


Ref: (Ref. Bailey and Love, - 1411)
Sol: A number of risk factors have been identified for chronic rejection of a kidney transplant. These are :
- Previous episodes of acute rejection;
- Poor HLA match;
- Long cold ischaemia time;
- Cytomegalovirus (CMV) infection;
- Raised blood lipids;
- Inadequate immunosuppression (including poor compliance).
The two most important risk factor for chronic rejection after kidney transplantation are acute rejection with vascular inflammation and
recurrent episodes of acute rejection.

Correct Answer. c

(112). In performing percutaneous gastrostomy, puncture site is selected over :

a. Cardia

b. Fundus

c. Body

d. Antrum

Solution. (c) Body


Reference – Read the text below
Sol:
- Percutaneous gastrostomy placement provides a more comfortable alternative to long term nasogastric feeding in patients who are
unable to maintain nutrition with oral intake.
- A puncture site is selected over the lower body of the stomach for this purpose.

Correct Answer. c

Copyright © 2014 Delhi Academy of Medical Sciences, All Rights Reserved. 51/93
(113). A clot size of a clenched fist is roughly equal to:

a. 200 ml

b. 300 ml

c. 400 ml

d. 500 ml

Solution. (d) 500 ml


Reference – Read the text below
Sol:
- A clot the size of a clenched fist is roughly equal to 500 ml.

Correct Answer. d

(114). Ideally the skin should be prepared for operation by:

a. Shaving 3 days before surgery

b. Shaving Immediately before surgery

c. Clippers

d. Depilation cream

Solution. (d) Depilation cream


Reference – Read the text below
Sol:
- The trauma of shaving undoubtedly results in lacerations to the skin that can increase the infection rate.
- It is preferable to use either clippers or, ideally, depilation cream, but the latter is expensive.

Correct Answer. d

(115). The most common indication for heart transplantation is:

a. Idiopathic cardiomyopathy

b. Valvular heart disease

c. Myocarditis

d. Congenital heart disease

Solution. (a) Idiopathic cardiomyopathy


Reference – Read the text below
Sol:
- The most common indications for heart transplantation are idiopathic cardiomyopathy and ischemic heart disease, but other indications
include valuvlar heart disease, myocarditis and congenital heart disease.

Correct Answer. a

Copyright © 2014 Delhi Academy of Medical Sciences, All Rights Reserved. 52/93
(116). Pierre robin syndrome does not included :

a. Cleft Lip

b. Cleft palate

c. Retrognathia

d. Glossoptosis

Solution. (a) Cleft Lip


Reference – Read the text below
Sol:
- Pierre robin sequence remains the most common syndrome.
- This syndrome comprises isolated cleft palate, retrognathia and a posteriorly displaced tongue (glossoptosis), which is associated with
early respiratory and feeding difficulties.

Correct Answer. a

(117). Parotid duct stones are :

a. Only radioopaque

b. Only radiolucent

c. Usually radioopaque

d. Usually radiolucent

Solution. (d) Usually radiolucent


Reference – Read the text below
Sol:
- Parotid duct stones are usually radiolucent and rarely visible on plain radiography. Sialography is needed to identify them.

Correct Answer. d

(118). Pneumaturia may be seen in :-

a. Prostatitis

b. Epididymoorchitis

c. After recent instrumentation

d. After a intravenous urogram

Solution. (c) After recent instrumentation


Reference – Read the text below
Sol:
- Pneumaturia refers to air in the voided urine.
- Pneumaturia may result from recent instrumentation of the bladder or from a fistula between the urinary tract and the intestine.

Correct Answer. c

Copyright © 2014 Delhi Academy of Medical Sciences, All Rights Reserved. 53/93
(119). The carcinomatous change in a long standing ulcer usually occurs at its:

a. Centre

b. Edge

c. Midway between centre and edge.

d. At any place in ulcer.

Solution. (b) Edge


Reference – Read the text below
Sol:
- Marjolin's ulcer: This is the eponym used to describe carcinomatous change occurring at the edge of a chronic venous ulcer.
- Such change can infact occur at the edge of any long standing benign ulcer.

Correct Answer. b

(120). During Psoas-hitch, care is taken for:

a. Ilioinguinal nerve

b. Genitofemoral nerve

c. Internal pudendal nerve

d. Perineal nerve

Solution. (b) Genitofemoral nerve


Reference – Read the text below
Sol:
- During lower third ureteric injury bladder is opend in midline or obliquely to allow mobilization of bladder to the ipsilateral psoas
muscle.
- It is sutured to the psoas muscle (psoas-hitch) but care is taken to avoid the genitofemoral nerve on the surface or femoral nerve deep
to psoas muscle.

Correct Answer. b

(121). Gritti – stokes amputation is:

a. Above knee

b. Through knee

c. Below knee

d. Through tarsal bones

Solution. (a) Above knee


Reference – Read the text below
Sol: Amputations – · Gritti- stokes amputation – transcondylar (femur) · Syme’s amputation – Tibia and fibula are sectioned as low as
possible to the top of the mortice joint. · The stump must be of sufficient length to give the required leverage (below the knee, not less
than 8 cm (preferable 10-12 cm); above the knee, not less than 20 cm). · A below knee amputation is much better than an above-knee (or
Gritti - stokes) amputation in them of eventual mobility. Every attempt should be made to preserve the knee joint if the extent of
ischaemia or trauma allow this.

Correct Answer. a

Copyright © 2014 Delhi Academy of Medical Sciences, All Rights Reserved. 54/93
(122). Treatment of desmoid tumor

a. Wide local excision

b. Local excision only

c. Wide local excision with radiotherapy

d. Chemotherapy

Solution. (a) Wide local excision


Reference – Read the text below
Sol:
- It is a locally invasive non metastatisisng fi brous tumor
- Occurs in mesentry or abdominal wall of patients undergoing colectomy
- 80%cases in woman
- Tumor composed of fi brous tissue containing multi nucleated plasmodialM masses resembling foreign body giant cell
- Very slow growth occurs
Tends to infi ltrate muscle in immediate area Treatment
- Tumor is excised widely with a surrounding margin of at least 2.5 cm of healthy tissue recurrence commonly takes place
- Repair of the defect in the abdominal wall with nylon mesh is required
- These tumors are moderately radiosensitive
- Now these are the lines from standard books no where its mentioned that whether wide local or local incision done

Correct Answer. a

(123). Most important source of NHBD donor are:

a. Category 1

b. Category 2

c. Category 3

d. Category 4

Solution. (c) Category 3


Ref: (Ref. Bailey and Love- 1419)
Sol: NBHD can be grouped according to the Maastricht classification as follows :
- Category 1 : dead on arrival at hospital;
- Category 2 : unsuccessful resuscitation in hospital;
- Category 3 : ‘awaiting cardiac arrest’ after withdrawal of support;
- Category 4 : cardiac arrest while brain dead.
Maastricht category 1 and 2 donors are sometimes referred to as uncontrolled NHBD. The warm ischaemic time of kidneys from these
two categories of donor is usually longer and less predictable than in the case of category 3 (controlled) donors. The majority of NHBD
organs used for transplantation are from category 3 donors.

Correct Answer. c

(124). A 36-year-old asian male complains of difficulty swallowing.Esophagoscopy reveals a polypoid mass that is subsequently biopsied. In
addition to tumor cells, the esophageal biopsy show normal smooth muscle and striated muscle in the same section. Which portion of the
esophagus was the source of this biopsy?

a. Lower esophageal sphincter

b. Lower third of the esophagus

c. Middle third of the esophagus

d. Upper esophageal sphincter

Solution. (c) Middle third of the esophagus


Reference – Read the text below
Sol:
- The muscularis of the upper third of the esophagus is composed entirely of striated muscle.
- The middle third contains both striated and smooth muscle.
- The lower third and lower esophageal sphincter contain only smooth muscle.
- There is no such thing as the upper esophageal sphincter.

Correct Answer. c

Copyright © 2014 Delhi Academy of Medical Sciences, All Rights Reserved. 55/93
(125). A 25-year-old woman with sickle cell anemia complains of steady pain in her right upper quadrant with radiation to the right shoulder,
especially after large or fatty meals. Her physician diagnoses gallstones. Of which of the following compounds are these stones most
likely composed?

a. Calcium bilirubinate

b. Calcium oxalate

c. Cholesterol

d. Cholesterol and calcium bilirubinate

Solution. (a) Calcium bilirubinate


Reference – Read the text below
Sol:
- Bilirubin is a degradative product of hemoglobin metabolism. Bilirubin (pigment) stones are specifically associated with excessive
bilirubin production in hemolytic anemias, including sickle cell anemia.
- Bilirubin stones can also be seen in hepatic cirrhosis and liver fluke infestation.
- Calcium oxalate stones and cystine stones are found in the kidney, rather than the gallbladder.
- Pure cholesterol stones are less common than mixed gallstones, but have the same risk factors, including obesity and multiple
pregnancies.
- Mixed stones are the common "garden variety" gallstones, found especially in obese, middle aged patients, with a female predominance

Correct Answer. a

(126). Which of the following best represents 'ranula'?

a. A type of epulis.

b. A thyroglossal cyst.

c. Forked uvula.

d. Extravasation cyst.

Solution. (d) Extravasation cyst


Ref – D-209
Sol: RANULA Extravasation cyst arising from sublingual gland or mucus glands of Nuhn or glands of Blandin in the floor of the mouth.
CLINICAL FEATURES OF RANULA
- Bluish swelling in floor of mouth
- Non-tender
- Fluctuants and cross fluctuant.
- Brilliantly transilluminant

Correct Answer. d

Copyright © 2014 Delhi Academy of Medical Sciences, All Rights Reserved. 56/93
(127). The treatment of choice for an 8 mm retained common bile duct (CBD) stone is :

a. Laproscopic CBD exploration.

b. Percutaneous stone extraction.

c. Endoscopic stone extraction.

d. Extracorporeal shock wave lithotripsy.

Solution. (c) Endoscopic stone extraction.


Reference – Sch-1201
Sol: CHOLEDOCHOLITHIASIS (BILE DUCT STONES)
- Most of these stones are formed in gall bladder.
- Incidence increases with age.
Investigations: Endoscopic Cholangiography – Gold Standard Treatment: A. Stones revealed on endoscopic cholangiogram
Sphincterotomy and ductal clearance of the stones followed by a laproscopic cholecystectomy. B. Retained or recurrent stones following
cholecystectomy are best treated endoscopically.
- Retained: Those diagnosed shortly after the cholecystectomy
- Recurrent: Those diagnosed month/year late

Correct Answer. d

(128). Reynold’s pentad of acute obstructive cholangitis includes all except

a. Jaundice

b. Shock

c. Altered mental Status

d. Intermittent pain.

Solution. (d) Intermittent pain


Reference – Sch-1201
Sol: Charcot ‘s triad of ascending cholangitis
- Intermittent pain
- Intermittent fever
- Intermittent Jaundice
Reynold’s pentad of acute Obstructive cholangitis
- Persistent Pain
- Persistent Fever
- Persistent Jaundice
- Shock
- Altered mental Status

Correct Answer. d

Copyright © 2014 Delhi Academy of Medical Sciences, All Rights Reserved. 57/93
(129). A 65-year-old miner has lost 7 kgs weight within two months, has presented with cough, and blood streaked sputum. He was treated for
pulmonary tuberculosis 10 years ago. He also has drooping of his left eyelid for one month. On physical examination, there is ptosis of
the left eye and pupillary miosis. Chest X-ray revealed round opacification in the left upper apical lobe. What is the most probable
diagnosis: -

a. Secondary tuberculosis

b. Adenocarcinoma

c. Squamous cell carcinoma

d. Asbestosis

Solution. (c) Squamous cells carcinoma


Reference – Read the text below
Sol:

Correct Answer. c

Copyright © 2014 Delhi Academy of Medical Sciences, All Rights Reserved. 58/93
(130). 45-year-old gentleman has undergone truncal vagotomy and pyloroplasty for bleeding duodenal ulcer seven years ago. Now he was having
intractable recurrent symptoms of peptic ulcer. All of the following suggest the diagnosis of Zollinger Ellison syndrome, except:

a. Basal acid output of 15 meq/hour

b. Serum gastrin value of 500 pg/ml

c. Ulcers in proximal jejunum and lower end of esophagus

d. Serum gastrin value of 200 pg/ml with secretin stimulation

Solution. (d) Serum gastrin value of 200 pg/ml with secretin stimulation
Reference – Read the text below
Sol: ZOLLINGER –ELLISON SYNDROME Severe peptic ulcer diathesis secondary to gastric acid hypersecretion due to unregulated gastrin release
from non- cell endocrine tumor (gastrinoma) defines the components of Zollinger –Ellison syndrome. Tumor Distribution: - 1. 80% within
gastrinoma triangle. 2. Duodenal tumors constitute the most common non-pancreatic lesion. 3. Lesser common sites – Stomach, bones, ovaries,
heart, Liver, Lymph Nodes. Clinical Manifestation: 1. Most common = Peptic Ulcer (> 90%). 2. Next most common – Diarrhoea Gastrinomas can
develop in the presence of MEN 1 syndrome.

Correct Answer. d

Copyright © 2014 Delhi Academy of Medical Sciences, All Rights Reserved. 59/93
(131). All familial conditions are associated with RCC except?

Copyright © 2014 Delhi Academy of Medical Sciences, All Rights Reserved. 60/93
a. Neurofibromatosis

b. VHL

Copyright © 2014 Delhi Academy of Medical Sciences, All Rights Reserved. 61/93
c. Birt Hogg Dube syndrome

d. Hereditary Papillary RCC

Solution. (b) VHL


Ref: Read the text below
Sol:

Correct Answer. b

Copyright © 2014 Delhi Academy of Medical Sciences, All Rights Reserved. 62/93
(132). Which of the following incision is wrongly matched?

a. Lazy S incision- Parotid

b. Subareolar Incision- Gynaecomastia

c. Chaveron Incision- Whipple’s operation

d. Rutherford Morrision- Renal transplant

Solution. (d) Rutherford Morrision- Renal transplant


Ref: Read the text below
Sol:

Correct Answer. d

Copyright © 2014 Delhi Academy of Medical Sciences, All Rights Reserved. 63/93
(133). All of the following extraintestinal manifestations of ulcerative colitis respond to colectomy except : -

a. Primary sclerosing cholangitis

b. Pyoderma gangrenosum

c. Episcleritis

d. Peripheral arthralgia

Solution. (a) Primary sclerosing cholangitis


Reference – Sabiston-1428
Sol: Extra Intestinal Manifestation of Ulcerative Colitis A. Most common extraintestinal manifestations of ulcerative colitis are rheumatologic
e.g peripheral arthritis/Ankylosing Spondylitis.

Correct Answer. a

(134). A 45-year-old builder fell from scaffolding to the ground this evening. He landed on his left side and initially had pain in his left lower
chest. He has now developed severe abdominal pain. He is becoming increasingly tachycardic and is hypotensive. His abdomen is tender
with guarding and he is tender over his left 10th and 11th ribs.Investigation of choice would be:

a. Diagnostic peritoneal tap or lavage.

b. CT abdomen ± contrast.

c. Erect chest X-ray.

d. Supine abdominal X-ray.

Solution. (a) Diagnostic peritoneal tap or lavage.


Ref:Read the text below
Sol:
- Lower chest injuries may be associated with rupture of liver or spleen.
- Splenic rupture may present immediately or after a delay of hours, even days.
- The signs are peritonism and shock that is out of proportion to the degree of observed blood loss or apparent trauma.
- The patients may be tender locally, in the left upper quardrant, or generally.
- Ultrasound or CT scan will show the splenic rupture but if the patient is shocked and there is likely to be any delay in arranging
imaging (e.g. outside of working hours), diagnostic peritoneal lavage will confirm the presence of intra-peritoneal haemorrhage.
- Lapartomy need not wait for imaging.

Correct Answer. a

Copyright © 2014 Delhi Academy of Medical Sciences, All Rights Reserved. 64/93
(135). False about cholecystectomy

a. Open cholecystectomy CBD injury rate is 0.1-0.2%

b. In Lap Cholecystectomy complication of CBD injury is double that of open

c. Abnormal GB, CBD, Cystic artery in 1/3 cases

d. Open cholecystectomy is the treatment of choice for cholelithiasis

Solution. (d) Open cholecystectomy is the treatment of choice for cholelithiasis


Ref:Read the text below
Sol:
- Still despite higher CBD injury rate Lap cholecystectomy is treatment of choice

Correct Answer. d

(136). All the following statements concerning popliteal artery aneurysms are true, except :

a. Approximately 50 percent are associated with aneurysms at other sites

b. Rupture into the popliteal space is a frequent complication

c. Associated thrombosis carries a high risk of amputation

d. Associated distal embolization may result in tissue loss

Solution. (b) Rupture into the popliteal space is a frequent complication


Ref: (Ref. . Schwartz, - 148
Sol:
- Popliteal aneurysms usually are associated with either distal embolization or thrombosis, both of which can cause limb-threatening
ischemia and gangrene.
- Although rupture is a serious threat with abdominal or thoracic aneurysms. With popliteal aneurysms, it occurs in only 5 to 10 percent
of reported series.
- Approximately 50 percent of popliteal aneurysms are associated with aneurysms else where in the body, particularly in the abdominal
aorta or the contralateral popliteal artery.
- Because these lesions typically are small, excision usually is not necessary, and bypass is acceptable when cambined with ligation of the
aneurysms.

Correct Answer. b

(137). Treatment of choice for post catheterization aneurysm of femoral artery is :

a. Spontaneous resolution

b. USG guided compression

c. Surgical repair

d. USG guided thrombi injection

Solution. (d) USG guided thrombi injection


Ref: Schwartz’s- 727
Sol:
- Postcatheterization groin pseudoaneurysms present with local tenderness, swelling, and the presence of a tender pulsatile mass.
- The diagnosis is confirmed with ultrasound scanning. Treatment depends on the size of the pseudoaneurysm and whether the patient is
anticoagulated.
- Spontaneous resolution is less likely if the patient is anticoagulated. Treatment has evolved from surgical repair of all
pseudoaneurysms, through ultrasound-guided compression to the current standard therapy, which is ultrasound-guided thrombin
injection.
- Surgical repair is reserved for those who fail injection or have a contraindication to thrombin injection such as wide neck, inability to
compress the neck, or presence of an arteriocvenous fistula.

Correct Answer. d

Copyright © 2014 Delhi Academy of Medical Sciences, All Rights Reserved. 65/93
(138). The most common site of lodgment of embolus is lower limb is :

a. Aorta-iliac junction

b. Ilion femoral junction

c. Common femoral bifurcation

d. Femoro – popliteal junction

Solution. (c) Common femoral bifurcation


Ref: Schwartz’s- 759)
Sol:
- Acute lower extremity ischemia manifests with the “five Ps”: pain, pallor, paresthesias, paralysis, and pulselessness, to which some add
a sixth “P” – poikilothermia ie “perishing cold”.
- Pain, however, is what causes a patient to present to the emergency room. The most common location for an embolus to lodge in the leg
is at the common femoral bifurcation. Typically a patient will complain of foot and calf pain, In addition to absent pulses, there is a
variable diminution of sensation, which varies from a mild reduction in sensation compared to the contralateral side to being completely
insensate.
- Inability to move the affected muscle group is a sign of very severe ischemia and necessitates urgent revascularization. During
evaluation of the affected extremity it is important to compare findings with the contralateral limb.
- Clinical evaluation is extremely important in determining the etiology and location of the obstruction. One of the most important pieces
of information to obtain is whether the patient has had prior vascular procedures or if there is a history of lower extremity claudication.

Correct Answer. c

(139). In patient of cleft palate, orthognathic surgery is done at :

a. 6 months

b. 1 ½ year

c. 16 year

d. 10 year

Solution. (c) 16 year


Ref: (Ref. Bailey and love- 669)
Sol:
- The principal dentofacial deformity associated with cleft lip and palate is underdevelopment in both the horizontal and vertical direction
of the maxilla.
- This leads to a pseudoprognathism in late adolescence, which is not correctable by orthodontic fixed-appliance therapy alone. Patients
needing orthognathic surgery can be identified as early as 10 years old, although planning and treatment does not commence until 14-15
years of age.
- Treatment with fixed appliances to align teeth in each dental arch is carried out over a period of 18-24 months as a prelude to
orthognathic surgery.
- Orthognathic surgery may require maxillary osteotomy advancement alone or bimaxillary osteotomy and genioplasty.
- Rigid fixation with or without bone grafting of the maxilla is essential as cleft lip and palate patients undergoing orthognathic surgery
have a high risk of a skeletal relapse as a result of the scaring associated with primary cleft lip and palate surgery.

Correct Answer. c

Copyright © 2014 Delhi Academy of Medical Sciences, All Rights Reserved. 66/93
(140). “Snowstorm appearance” is seen in:

a. HIV associated parotitis

b. Recurrent parotitis

c. Candida parotitis

d. Pleomorphic adenoma

Solution. (b) Recurrent parotitis


Ref: Bailey & Love, -760
Sol: Recurrent parotitis of childhood
- Recurrent parotitis of childhood is a distinct clinical entity of unknown aetiology and variable prognosis. It is characterised by rapid
swelling of one or both parotid glands, in which the symptoms are made worse by chewing and eating.
- Systemic upset with fever and malaise is variable. The symptoms usually last from 3 to 7 days, and are then followed by a quiescent
period of weeks to several months. Children usually present between the ages of 3 and 6 years, although symptoms have been reported
in infants as and young as 4 months.
- The diagnosis is based on the characteristic history and can be confirmed by sialography. This shows a characteristic punctate
sialectasis likened to a ‘snowstorm’. The condition usually responds to short courses of antibiotics although, if recurrence is frequent,
prophylactic low-dose antibiotics may be required for several months or even years.
- Few children require formal parotidectomy. However, if the onset of symptoms is late, e.g adolescence/ early adulthood, fewer patients
respond to conservative measures and may require total conservative parotidectomy.

Correct Answer. b

(141). Proteus mirabilis is associated with which renal stone?

a. Calcium oxalate

b. Uric acid

c. Phosphate

d. Struvite

Solution. (d) Struvite


Ref: Read the text below
Sol:
- Struvite stones are also known as infection stones, The formation of struvite stones is associated with the presence of urea-splitting
bacteria, most commonly Proteus mirabilis (but also Klebsiella, Serratia, Providencia species).
- These organisms are capable of splitting urea into ammonia, decreasing the acidity of the urine and resulting in favorable conditions for
the formation of struvite stones.
- Struvite stones are always associated with urinary tract infections

Correct Answer. d

(142). Which of the following does not increase the risk of malignancy in the male genital organs?

a. Herpes virus infection

b. Human papillomavirus infection

c. Bowen's disease

d. Cryptorchidism

Solution. (a) Herpes virus infection


Ref: Read the text below
Sol:
- Herpes virus causes benign infection.It is not premalignant.

Correct Answer. a

Copyright © 2014 Delhi Academy of Medical Sciences, All Rights Reserved. 67/93
(143). 26 year old male has acute left loin pain radiating to the left groin. Urinalysis reveals puria. He undergoes an intravenous urogram which
reveals collections of contrast medium in dilated papillary collecting ducts of both the right and left kidneys. Both kidneys are enlarged.
The most likely diagnosis is

a. Dilatation of the ureter

b. Horse-shoe kidney

c. Medullary sponge kidney

d. Renal tuberculosis

Solution. (d) Renal tuberculosis


Ref: Read the text below
Sol:
- Renal involvement may be indolent, with a latency period of more than 20 years after the primary infection to the appearance of urinary
tract symptoms of hematuria and stone disease. In patients with renal tuberculosis, treatment involves antitubercular drugs, with
surgical excision as an adjunct to antitubercular therapy.
- The urine can be free of bacteria in less than 72 hours, but anatomic changes can progress as part of the healing process.Sterile pyuria
is characterstic. Females with genital tuberculosis may present with infertility, menstrual disorders, and pain. Pregnancy is unusual in
the presence of genital tuberculosis.
- When pregnancy occurs, spontaneous abortion or ectopic pregnancy usually result. As a result of the lack of clinical features, diagnosis
of genital tuberculosis may be difficult.

Correct Answer. d

(144). Which of the following statements does not correctly describe the suprarenal gland ?

a. Foetal glands are larger than the foetal kidney

b. In ectopia of kidney, the suprarenal gland is also in an ectopic position lying on the upper pole of ectopic kidney

c. Cortex of suprarenal gland develops from coelomic epithelium

d. Three suprarenal arteries enter the gland peripherally and a single vein emerges from the hilum

Solution. (b) In ectopia of kidney, the suprarenal gland is also in an ectopic position lying on the upper pole of ectopic kidney
Ref: Read the text below
Sol:
- Even in cases of ectopia of kidney the suprarenal gland lies in its normal position.

Correct Answer. b

(145). Diaphragmatic hernia can occur through all the following, except:

a. Oesophageal opening

b. Costovertebral triangle

c. Costal and sternal attachment of the diaphragm

d. Inferior vena caval opening

Solution. (d) Inferior vena caval opening


Ref: Read the text below
Sol: Inferior vena caval opening is situated at about 8th thoracic vertebra, transmits the vena cava and is not a site for diaphragmatic
herniation. Possible sites of diaphragmatic hernia are:
1. Oesophageal hiatus: Site of hiatus hernia, comprising 98% of all diaphragmatic hernias.
2. Sternocostal triangle: Lies between sternal and costal areas and transmits superior epigastric vessels and is a potential site for
acquired herniation.
3. Lumbosacral triangle: Situated between costal and lumbar vertebral portions (costovertebral). Stomach and other peritoneal viscera
herniate through it.

Correct Answer. d

Copyright © 2014 Delhi Academy of Medical Sciences, All Rights Reserved. 68/93
(146). Microvascular fibular free flap is raised on:

a. Ant tibial artery

b. Posterior tibial artery

c. Peroneal artery

d. Dorsalis pedis

Solution. (c) Peroneal artery


Ref: Bailey & Love, - 747)
Sol:

Correct Answer. c

(147). All of the following favours non-functioning endocrine pancreatic tumours over Ca pancreas except :

a. Size < 5 cm

b. Calcification

c. Chromogranin A positive

d. None of the above

Solution. (a) Size < 5 cm


Ref:Bailey and Love- 820)
Sol: Differences between pancreatic cancer and non-functioning endocrine pancreatic tumours (NF-PETs)

Correct Answer. a

Copyright © 2014 Delhi Academy of Medical Sciences, All Rights Reserved. 69/93
(148). Emergency Room thoractomy is considered futile in all of the following except:

a. Blunt trauma with no sign of life at scene

b. CPR in absence endotracheal incubation for > 5 mins.

c. CPR for > 10 mins

d. Patient in extreme is with falling BP despite volumes resuscitations (BP syst > 60 mmHg.)

Solution. (d) Patient in extreme is with falling BP despite volumes resuscitations (BP syst > 60 mmHg.) Ref: Bailey & Love- 345
Sol: Emergency room thoractomy : ERT should be reserved for those patients suffering penetrating injury in whom signs of life are still present.
Patients who have received cardiopulmonary resuscitation (CPR) in the prehospital phase of their care are unlikely to survive, and electrical activity
must be present. The survival rates for ERT in patients with penetrating trauma in whom the blood pressure is falling despite adequate
resuscitation are shown in table. It is important to make a distinction between :
 ERT for the control of haemorrhage or tamponade or in some situations, for internal cardiac massage;
- Urgent planned thoracotomy for definitive correction of the problem – this sually takes place preferably in the more controlled environment of the
operating theatre.
In certain situations, ERT is considered futile :
- CPR in the absence of endotracheal intubation for more than 5 min.
- CPR for more than 10 min (with or without endotracheal intubations).
- blunt trauma when there have been no signs of life at scene.

Correct Answer. d

Copyright © 2014 Delhi Academy of Medical Sciences, All Rights Reserved. 70/93
(149). Indications for damage control surgery are all of the following except :

a. Complex abdominal injury

b. Patient in Hypothermia with acidosis and coagulopathy

c. Operating time < 60 mins

d. Inability to achieve hemostasis

Solution. (c) Operating time < 60 mins


Ref: (Ref. Bailey & Love, - 351)
Sol:

Correct Answer. c

Copyright © 2014 Delhi Academy of Medical Sciences, All Rights Reserved. 71/93
(150). NICE Guides for CT scan in head injury is all, except :

a. GCS < 13 at any point

b. GCS of 14 at 2 hr.

c. Suspected skull base fracture

d. At least 2 episodes of vomiting

Solution. (d) At least 2 episodes of vomiting


Ref: Read the text below
Sol:

Correct Answer. d

(151). False about Malakoplakia is :

a. Predominantly in males

b. Most common site is bledder

c. CT scan shows hyporasuclar

d. None of the above

Solution. (a) Predominantly in males


Ref: Smith’s Urology- 252)
Sol:
- Malacoplakia is a disease that predominantly affects women, a fact largely reflected by its association with UTIs. Most patients are over
50 years old. Although these tumors can form throughout the body, 75% occur in the urinary tract, and within the urinary tract 70% of
cases are found in the bladder, 15% in the upper tracts, and 15% in both.
- The clinical presentation of malacoplakia varies with the site of involvement. Concurrent systemic illness or an otherwise debilitating
condition are not infrequent. Bladder involvement is suggested by hematuria and irritative voiding symptoms. Renal lesions may present
with colicky flank pain, fever, and a flank mass.
- When renal involvement is bilateral, as in about 50% of cases (Stanton and Maxted, 1981), presentation with azotemia or uremia is not
uncommon. In general, the symptoms of malacoplakia can mimic those of several other, more common UTIs, as well as genitourinary
malignancy.
- In patients with lower tract symptoms, cystoscopy usually reveals mucosal plaques or nodules. As the disease progresses, these lesions
can become large, fungating masses. At this point, IVU may reveal filling defects in the affected portion of the collecting system. The
inflammatory mass may also cause ureteral obstruction and hydronephrosis.
- On ultra-sonography, multifocal renal involvement may display as renal enlargement and a general increase in parenchymal
echotexture (Hartman et al, 1980). On CT, hypodense parenchymal masses may be seen focally or diffusely; CT can also demonstrate any
extension outside the urinary tract (Hartman, 1985). Angiography generally shows a hypovascular mass with peripheral neovascularity.

Correct Answer. a

Copyright © 2014 Delhi Academy of Medical Sciences, All Rights Reserved. 72/93
(152). In order to expose the celiac axis, left renal artery, superior mesenteric artery and abdominal aorta in a case of trauma, which of the
following is required?

a. Left medial visceral rotation

b. Right medial visceral rotation

c. Cranial visceral rotation

d. Caudal visceral rotation

Solution. (a) Left medial visceral rotation


Ref:Read the text below
Sol: Schwartz's Principles of Surgery, Ninth Edition
- The aorta, celiac axis, proximal superior mesenteric artery (SMA), and left renal arteries can be exposed with a left medial visceral
rotation
- This is done by incising the lateral peritoneal reflection (white line of Toldt) beginning at the distal descending colon and extending the
incision along the colonic splenic flexure, around the posterior aspect of the spleen, and behind the gastric fundus, ending at the
esophagus.
- The left colon, spleen, pancreas, and stomach are then rotated toward the midline.
- The authors prefer to leave the kidney in situ when mobilizing the viscera because this exaggerates the separation of the renal vessels
from the SMA.
- Proximal control of the aorta is obtained at the diaphragmatic hiatus; if an aortic injury is supraceliac, transecting the left crus of
diaphragm or performing left thoracotomy may be necessary.
- Inferior vena cava injuries are approached by a right medial visceral rotation
- Proximal control is obtained just above the aortic bifurcation with direct pressure via a sponge stick; the injury is identified by cephalad
dissection along the anterior surface of the inferior vena cava.
- The operative approach for SMA injuries is based on the level of injury.
- Fullen zone I SMA injuries, located posterior to the pancreas, can be exposed by a left medial visceral rotation.
- Fullen zone II SMA injuries, extending from the pancreatic edge to the middle colic branch, are approached via the lesser sac along the
inferior edge of the pancreas at the base of the transverse mesocolon; the pancreatic body may be divided to gain proximal vascular
access.
- More distal SMA injuries, Fullen zones III and IV, are approached directly within the mesentery.
- A venous injury behind the pancreas, from the junction of the superior mesenteric, splenic, and portal veins, is accessed by dividing the
neck of the pancreas.
- Injuries of the iliac vessels pose a unique problem for emergent vascular control due to the number of vessels, their close proximity, and
cross circulation.
- Proximal control at the infrarenal aorta arrests the arterial bleeding and avoids splanchnic and renal ischemia; however, venous injuries
are not controlled with aortic clamping.
- Tamponade with a folded laparotomy pad held directly over the bleeding site usually will establish hemostasis sufficient to prevent
exsanguination.
- If hemostasis is not adequate to expose the vessel proximal and distal to the injury, sponge sticks can be strategically placed on either
side of the injury and carefully adjusted to improve hemostasis. Alternatively, complete pelvic vascular isolation may be required to
control hemorrhage for adequate visualization of the injuries.
- The right common iliac artery obscures the bifurcation of the vena cava and the right iliac vein; the iliac artery can be divided to expose
venous injuries of this area
- The artery must be repaired after the venous injury is treated, however, because of limb-threatening ischemia
- Once overt hemorrhage is controlled, sources of enteric contamination are identified by serially running along the small and large
bowel, looking at all surfaces.
- Associated hematomas should be unroofed to rule out adjacent bowel injury.

Correct Answer. a

Copyright © 2014 Delhi Academy of Medical Sciences, All Rights Reserved. 73/93
(153). Most immediate complication of ileostomy in the post-operative period

a. Obstruction

b. Necrosis

c. Diarrhea

d. Prolapse

Solution. (b) Necrosis


Ref:Read the text below
Sol: Complications of Ileostomy
- Stoma necrosis may occur in the early postoperative period and usually is caused by skeletonizing the distal small bowel and/or
creating an overly tight fascial defect.
- Limited mucosal necrosis above the fascia may be treated expectantly, but necrosis below the level of the fascia requires surgical
revision.
- Stoma retraction may occur early or late, and may be exacerbated by obesity.
- Local revision may be necessary.
- The creation of an ileostomy bypasses the fluid absorbing capability of the colon, and dehydration with fluid and electrolyte
abnormalities is not uncommon.
- Ideally, ileostomy output should be maintained at less than 1500 mL/d to avoid this problem.
- Bulk agents and opioids (Lomotil, Imodium, tincture of opium) are useful.
- The somatostatin analogue,
- Octreotide, has been used with variable success in this setting.
- Skin irritation also can occur, especially if the stoma appliance fits poorly. Skin protecting agents and custom pouches can help to solve
this problem.
- Obstruction may occur intra-abdominally or at the site where the stoma exits the fascia.
- Parastomal hernia is less common after an ileostomy than after a colostomy, but can cause poor appliance fitting, pain, obstruction, or
strangulation.
- In general, symptomatic parastomal hernias should be repaired.
- Repair usually requires re-siting the stoma to the contralateral side of the abdomen.
- Prolapse is a rare, late complication and often is associated with a parastomal hernia.
- Valve slippage resulting in either leakage or obstruction is a common complication of a continent Kock pouch ileostomy

Correct Answer. b

(154). Most common tumor of hand is?

a. Enchondroma

b. Chondroblastoma

c. Giant cell tumour

d. Osteoid osteoma

Solution. (a) Enchondroma


Ref.: Read the text below
Sol :
- Most common tumour of hand : Ganglion > Enchondroma
- Most common bony tumour of hand : Enchondroma
- Most common soft tissue tumour of hand :Ganglion
- Most common malignant bone tumour in hand : Chondrosarcoma

Correct Answer. a

Copyright © 2014 Delhi Academy of Medical Sciences, All Rights Reserved. 74/93
(155). Coronary graft is most commonly taken form?

a. Femoral vein

b. Saphenous vein

c. Axillary vein

d. Cubital vein

Solution. (b) Saphenous vein


Ref.: Read the text below
Sol :
- Saphenous vein grafts is probably the most common surgical reconstruction performed for correcting coronary artery disease
- “Saphenous veins are the most commonly employed conduits in coronary revascularization”

Correct Answer. b

(156). Which of the following bacterial meningitis is associated with subdural effusion?

a. H. Influenza

b. Neisseria meningitis

c. Streptococcus pneumonia

d. Enterococcus

Solution. (a) H. Influenza


Ref.: Read the text below
Sol :
- The most serious manifestation of infection with Hib is meningitis which primarily affects children <2 years of age .
- Fever and altered central nervous system function are the most common features.
- Subdural effusion, the most common complication, is suspected when, despite 2 or 3 days of appropriate antibiotic therapy, the infant
has seizures, hemi paresis, or continued obtundation.
- The overall mortality rate from Hib meningitis is 5%, and the morbidity rate is high.
- Of survivors, 6% have permanent sensorineural hearing loss, and about one-fourth have a significant handicap of some type. If more
subtle handicaps are sought, up to half of survivors are found to have some neurologic squeal, such as partial hearing loss and delayed
language development.

Correct Answer. a

(157). Limb salvage can be done in all except:

a. Nerve injury

b. Vascular injury

c. Bone injury

d. Muscle injury

Solution. (b) Vascular injury


Ref.: Read the text below
Sol :
- Limb salvage cannot be done in cases of vascular injury, this is further explained by indications for amputations

Correct Answer. b

Copyright © 2014 Delhi Academy of Medical Sciences, All Rights Reserved. 75/93
(158). An ABG analysis shows: pH 7.2, raised pCO2, decreased HCO3; diagnosis is:

a. Respiratory acidosis

b. Compensated metabolic acidosis

c. Respiratory and metabolic acidosis

d. Respiratory alkalosis

Solution. (c) Respiratory and metabolic acidosis


Ref.: Read the text below
Sol :

Correct Answer. c

(159). A 42-year-old homeless man is brought in to casualty by the police. He was found wandering the streets claiming to have been assaulted.
He denies any loss of consciousness. He has a large scalp laceration over his occiput. He alert and orientated and has no focal
neurological signs. He smells strongly of alcohol. Next step would be;

a. Skull X-rays

b. Urgent CT scan.

c. Admit for 24 hrs observation.

d. Discharge home.

Solution. (a) Skull X-rays


Ref:Read the text below
Sol:
- Skull x-ray should be performed, in the first instance, for three reasons.
- He has a significant scalp laceration, he has been drinking and he may have been assaulted (so the X-rays may be useful from a forensic
point of view).
- He should be admitted overnight, as he will be unobserved if he is discharged.
- There is a higher incidence of extradural and subdural haemorrhage in alcoholics.
- Misdiagnosis is common due to difficulties in assessing patients who are confused or inebriated.

Correct Answer. a

Copyright © 2014 Delhi Academy of Medical Sciences, All Rights Reserved. 76/93
(160). Which of the following is not true pancreatic cysts:

a. Associated with Von Hippel-Lindau Disease in 50%

b. It is associated with cysts of liver and kidney

c. Most go on to be associated with chronic pancreatitis

d. Pancraetic cysts are very rare.

Solution. (c) Most go on to be associated with chronic pancreatitis


Ref:Read the text below
Sol:
- Solitary cysts of the pancreas are rare. In contrast, multiple pancreatic cysts, lined with cuboidal epithelium, are more common.
- They are frequently associated with polycystic disease of the liver and/or kidney, and they can be seen in up to half of patients with von
Hippel-Lindau syndrome.
- Pancreatic cysts only rarely become symptomatic and, in general, no treatment is indicated.

Correct Answer. c

(161). Management of leukoplakia of the oral cavity includes

a. Excisional biopsy of all lesions

b. Application of topical antibiotics

c. Low-dose radiation therapy

d. Ascertaining that dentures fit properly

Solution. (d) Ascertaining that dentures fit properly


Ref:Read the text below
Sol:
- White patches in the oral cavity (leukoplakia) sometimes are incorrectly interpreted as a pre-malignant condition.
- Microscopic examination of leukoplakia may in fact reveal hyperplasia, keratosis, or dyskeratosis, of which the last finding is the most
serious because of its association with malignancy.
- Only about 5% of patients with leukoplakia develop cancer. A suggested treatment protocol for patients with thin lesions advocates a
program of strict oral hygiene and avoidance of alcohol and tobacco.

Correct Answer. d

Copyright © 2014 Delhi Academy of Medical Sciences, All Rights Reserved. 77/93
(162). The most common diagnostic modality for prostate cancer is currently: -

a. Transrectal ultrasonography alone

b. PSA and digital rectal examination (DRE)

c. CT scan

d. Transrectal ultrasonography with biopsies

Solution. (d) Transrectal ultrasonography with biopsies


Sol: TRANSRECTAL ULTRASONOGRAPHY AND BIOPSIES
- The most common diagnostic modality for prostate cancer is currently transrectal ultrasonography (TRUS) with guided biopsies .
- TRUS provides imaging of the prostate and seminal vesicles using a 7.5mHz biplane intra-rectal probe measuring approximately 1.5cm
in diameter..
- Transrectal ultrasonography can image the outline of the prostate, cysts, abscesses, and calcifications within the prostate.

Correct Answer. d

(163). A 40-year-old female patient presented with dysphagia to both liquids and solids and regurgitation for 3 months. The dysphagia was non-
progressive. Manometric findings confirms the aperistalsis in the esophageal body. What is the most likely diagnosis: -

a. Carcinoma of the esophagus

b. Nutcracker Esophagus

c. Achalasia cardia

d. Reflux esophagitis with esophageal stricture

Solution. (c) Achalasia cardia


Sol: ACHALASIA CARDIA It is failure of relaxation of cardia (esophagogastric junction) due to disorganised oesophageal peristalsis, as a
result of failure of integration of parasympathetic impulses causing functional obstruction. Clinical Features
- Females between 20-40 years age group
- Presents with progressive dysphagia which is more for liquid than to solid food.
- Regurgitation and recurrent pneumonia are common.
(Non progressive dysphagia in this case does not rule the diagnosis) Manometric characteristics:
- Incomplete lower esophageal sphincter (LES) relaxation
- Aperistalasis in the esophageal body.
- Elevated LES pressure
- Increased intraesophageal baseline pressure relative to gastric baselines.
Esophagoscopy is done to confirm the diagnosis and to rule out carcinoma esophagus. Treatment Surgery: 1. Modified Heller's
operation( Oesophagocardiomyotomy.) 2. Negus hydrostatic dilatation. 3. Laproscopic or thoracoscopic cardiomyotomy

Correct Answer. c

Copyright © 2014 Delhi Academy of Medical Sciences, All Rights Reserved. 78/93
(164). Nerves that can be invaded directly by the malignant tumours of tongue are: -

a. Glossopharyngeal and Spinal accessory

b. Lingual and Hypoglossal

c. Spinal accessory and Vagus

d. Only vagus

Solution. (b) Lingual and Hypoglossal


Sol; Tumors of the tongue
1. Tumors of the tongue begin in the stratified epithelium of the surface and eventually invade into the deeper muscular structures.
2. The presentation is commonly an ulcerated or exophytic mass.
3. The regional lymphatics of the oral cavity are to the submandibular space and the upper cervical lymph nodes.
4. The lingual nerve and the hypoglossal nerve may be invaded directly by tumors.

Correct Answer. b

(165). 27 yrs old mahesh presented in surgical emergency following a road side accident with gross hematuria. Radiological evaluation
confirmed the renal cortical laceration < 1cm without any urinary extravasation.According to staging system developed by the American
Association for the surgery of trauma ,patient had trauma of:

a. Grade 1

b. Grade 2

c. Grade 3

d. Grade 4

Solution. (b) Grade 2


Reference – Sch-1544
Sol: American Association for the surgery of trauma SURGERY OF TRAUMA Grade Description of injury 1 Contusion or nonexpanding
subcapsular hematoma No laceration
2 Nonexpanding perirenal hematoma Cortical laceration < I cm deep without extravasation
3 Cortical laceration > I cm without urinary extravasation
4 Laceration: through corticomedullary junction into collecting system or Vascular: segmental renal artery or vein injury with contained
hematoma
5 Laceration: shattered kidney or Vascular: renal pedicle injury or avulsion

Correct Answer. b

(166). Which of the given option for enteral feeding access can be used for 12-24 months:

a. Nasogastric tube

b. Nasoduodenal tube

c. Nasojejunal tube

d. Percutaneous endoscopicgastrostomy

Solution. (d) Percutaneous endoscopicgastrostomy


Sol. OPTIONS FOR ENTERAL FEEDING ACCESS Nasogastric tube Short-term use only; aspiration risks;nasopharyngeal trauma;
frequent dislodgment Nasoduodenal/nasojejunal Short-term use; lower aspiration risks in jejunum; placement challenges (radiographic
assistance often necessary) Percutaneous endoscopicgastrostomy (PEG) Endoscopy skills required; may be used for gastric
decompression or bolus feeds; aspiration risks; can last 12–24 months; slightly higher complication rates with placement and site leaks

Correct Answer. d

Copyright © 2014 Delhi Academy of Medical Sciences, All Rights Reserved. 79/93
(167). Most common late complication of a colostomy is :

a. Colostomy retraction

b. Obstruction

c. Parastomal hernia.

d. Prolapse

Solution. (c) Parastomal hernia


Sol. COMPLICATIONS OF COLOSTOMY
- Colostomy necrosis
- Colostomy retraction (less problematic with a colostomy than with an ileostomy because the stool is less irritating to the skin than
succus entericus)
- Obstruction (intraabdominal or at the site where the bowel exits the fascia)
- Parastomal hernia is the most common late complication of a colostomy
and requires repair if it is symptomatic. Repair usually requires resiting the stoma to the contralateral side of the abdomen.
-Prolapse ( rarest)

Correct Answer. c

(168). Four patterns of benign calcification are common: diffuse, solid, central, and laminated or “pop-corn." Granulomatous infections such as
tuberculosis can demonstrate all except :

a. Diffuse

b. Solid

c. Central

d. Laminated or “pop-corn."

Solution. (d) Laminated or “pop-corn."


Sol.
- Calcification within a nodule suggests a benign lesion.
- Four patterns of benign calcification are common: diffuse, solid, central, and laminated or “pop-corn."
-Granulomatous infections such as tuberculosis can demonstrate the first
three patterns, whereas the popcorn pattern is most common in hamartomas

Correct Answer. d

(169). A 47-year-old woman who noticed a lump at the base of her neck was found to have a 2-cm nodule in the right lower lobe of the thyroid
gland. Fine-needle aspiration showed follicular cells. The next step in management should be:

a. CT scan of the neck and chest

b. MRI of the head and neck

c. Excisional nodulectomy

d. Hemithyroidectomy

Solution. (d) Hemithyroidectomy


Ref:Read the text below
Sol
- The presence of follicular cells on a fine-needle aspiration biopsy of a thyroid mass is a vexing problem.
- Because these cells may indicate either benign follicular adenoma or follicular carcinoma, it may not be possible to arrive at the
diagnosis preoperatively.
- The initial surgical approach—hemithyroidectomy versus total thyroidectomy— for a tumor with follicular cytology is a subject of
debate.

Correct Answer. d

Copyright © 2014 Delhi Academy of Medical Sciences, All Rights Reserved. 80/93
(170). A 36-year-old women presented with very severe left-sided abdominal pain. Her husband states that she has been pacing around the
bedroom all night, unable to find a comfortable position, and the patient describes the pain as being ‘worse than a labour pain’.Diagnosis
is

a. Ureteric pain.

b. Acute salpingitis.

c. Perforated peptic ulcer.

d. Pancreatitis.

Solution. (a) Ureteric pain.


Ref– Read the text below
Sol:
- The pain of ureteric colic is very severe and will often be described as the worse pain a patient has ever experienced.
- The characteristic restlessness and inability to find a position that is highly suggestive of this condition.
- A dipstick urine test will usually be positive for blood, and a ureteric calculus will sometimes be seen on the KUB film.

Correct Answer. a

(171). A 35 year old man presents with a 3 week history of epigastric pain which is worse prior to meals and at night. He has no other
symptoms. You elicit a mildly tender epigastrium on palpation; otherwise examination is normal. You suspect peptic ulcer disease.With
regards to Peptic Ulcer Disease (PUD):

a. Duodenal ulcers are 10 times more common than gastric ulcers

b. The most appropriate initial investigation is testing for Helicobacter pylori

c. This patient requires urgent endoscopy

d. Duodenal ulcers are more common in patients with blood group A

Solution. (b) The most appropriate initial investigation is testing for Helicobacter pylori
Reference – Read the text below
Sol:
- Duodenal ulcers are 2-3 times more common than gastric ulcers.
- Testing and treatment for Helicobacter pylori is the most appropriate initial step in this case.
- Peptic ulcer disease is more common in smokers. Duodenal ulcers are more common in patients with blood group O.

Correct Answer. b

(172). Gastric cancer:

a. When shown to be an adenocarcinoma on histology, is associated with Helicobacter pylori

b. Is usually a squamous cell carcinoma

c. Usually has a good prognosis

d. Most commonly presents with haematemesis

Solution. (a) When shown to be an adenocarcinoma on histology, is associated with Helicobacter pylori Reference – Read the text below
Sol:
- Gastric cancer is most commonly an adenocarcinoma. It has a poor prognosis due to the usual late presentation of disease.
- The most common symptom is epigastric pain.
- Diagnosis is usually made on either gastroscopy or barium meal.

Correct Answer. a

Copyright © 2014 Delhi Academy of Medical Sciences, All Rights Reserved. 81/93
(173). Subcapsular nephrectomy is indicated in –

a. Perinepric abscess

b. Hydronephrosis

c. Pyonephrosis

d. Solitary adenocarcinoma

Solution. (c) Pyonephrosis


Ref: Read the text below
Sol :
- Indications of subcapsular nephrectomy.
- Chronic renal infection.
- Previous renal surgery.
- Nephrectomy of transplanted kidney.

Correct Answer. c

(174). 58 years old male presenting with acute onset of varicocele on left side, most prbobable cause

a. Ca testes

b. Epididymitis

c. Inguinal lymphnodes

d. Ca kidney

Solution. (d) Ca kidney


Ref: Read the text below
Sol :
- 98% of idiopathic varicoceles occur on the left side, apparently because the left testicular vein connects to the renal vein (and does so at
a 90-degree angle), while the right testicular vein drains at less than 90-degrees directly into the significantly larger inferior vena cava.
Isolated right sided varicoceles are rare, and should prompt evaluation for an abdominal/ pelvic mass or a kidney cancer.
- A secondary varicocele is due to compression of the venous drainage of the testicle. A pelvic or abdominal malignancy is a definite
concern when a right-sided varicocele is newly diagnosed in a patient older than 40 years of age.
- One non-malignant cause of a secondary varicocele is the so-called "Nutcracker syndrome", a condition in which the superior
mesenteric artery compresses the left renal vein, causing increased pressures there to be transmitted retrograde into the left
pampiniform plexus.
- The most common cause is renal cell carcinoma (hypernephroma) followed by retroperitoneal fibrosis or adhesions

Correct Answer. d

(175). Renal trauma is best treated by –

a. Observation and supportive measures

b. Early drainage of perirenal haematoma

c. Heminephrectomy

d. Nephrostomy

Solution. (a) Observation and supportive measures


Ref: Read the text below
Sol :
- Most blunt renal injuries are low-grade; therefore, they are usually amenable to treatment with observation and bed rest alone.
- Penetrating trauma is more likely to be associated with more severe renal injury, thus requiring a higher index of clinical awareness.
- Further, penetrating trauma is more often associated with other abdominal injuries requiring laparotomy, thus providing the
opportunity for intraoperative renal staging and/or repair.
- Patients with indications for emergent exploration include those with hemodynamic instability. Expanding hematomas or active
hemorrhage suggests the possibility of high-grade renal injury. Patients with penetrating trauma who are stable and do not require
urgent laparotomy for other possible intra-abdominal injuries may be observed without immediate renal exploration

Correct Answer. a

Copyright © 2014 Delhi Academy of Medical Sciences, All Rights Reserved. 82/93
(176). Pseudo kidney is

a. Thickened bowel loop on USG

b. Hydronephrosis

c. Undescended kidney

d. Undescended testes

Solution. (a) Thickened bowel loop on USG


Ref: Read the text below
Sol :
- The pseudokidney of intussusception refers to the longitudinal ultrasound appearance of the intussuscepted segment of bowel.
- The fat containing mesentary which is dragged into the intussusception, containing vessels, is reminiscent of the renal hilum, with the
renal parenchyma formed by the oedematous bowel.

Correct Answer. a

(177). Thimble bladder is seen :

a. Acute tuberculosis

b. Chronic tuberculosis

c. Neurogenic bladder

d. Schistosomiasis

Solution. (b) Chronic tuberculosis


Ref: Read the text below
Sol :
Kidneys
- The most common finding in tuberculous cystitis is reduced bladder capacity.
- Typically, tuberculous cystitis manifests as a shrunken bladder with wall thickening. Occasionally, filling defects due to multiple
granulomas may also be seen. In advanced disease, the bladder may be diminutive and irregular (thimble bladder)
- Advanced bladder involvement may be complicated by vesicoureteral reflux due to fibrosis involving the ureteral orifice.
- Calcification of the bladder wall is rarely seen
- Intravenous urogram demonstrates a thickened, contracted, low-capacity bladder (thimble bladder)

Correct Answer. b

(178). A 60 year old female presented with hematuria and diagnosed transitional cell carcinoma of bladder stage TINIMo. Best treatment
modality is :

a. Transuretheral resection

b. Transuretheral resection and intravesical chemoimmunotherapy

c. Total cystectomy and pelvic lymphadenectomy

d. Systemic chemotherapy

Solution. (b) Transuretheral resection and intravesical chemoimmunotherapy


Ref: Read the text below
Sol :
- Transitional cell carcinoma (TCC) can be very difficult to treat. Treatment for limited stage TCC is surgical resection of the tumor, but
reoccurrence is common.
- Chemotherapy for TCC consists of the MVAC regimen (methotrexate, vinblastine, adriamycin and cisplatin).
- TCC can also be treated with infusions of BCG into the bladder

Correct Answer. b

Copyright © 2014 Delhi Academy of Medical Sciences, All Rights Reserved. 83/93
(179). Testicular cancer is common in

a. Ectopic testis

b. Undescended abdominal testis

c. Atrophic testis

d. Anteverted testis

Solution. (b) Undescended abdominal testis Ref: Read the text below Sol : The exact cause of testicular cancer is unknown. There is no
link between vasectomy and testicular cancer. Factors that may increase a man's risk for testicular cancer include:
- Abnormal testicle development
- History of testicular cancer
- History of undescended testicle
- Klinefelter syndrome Other possible causes include exposure to certain chemicals and HIV infection. A family history of testicular
cancer may also increase risk. Testicular cancer is the most common form of cancer in men between the ages of 15 and 35. It can occur
in older men, and rarely, in younger boys.

Correct Answer. b

(180). A 65-year-old man has undergone a hernia repair as a day- case. He has not passed urine since the procedure and on examination a
suprapubic mass is palpable.Most susceptible cause would be.

a. Acute urinary retention.

b. Blocked catheter.

c. Intravascular depletion.

d. Renal impairment.

Solution. (a) Acute urinary retention.


Ref– Read the text below
Sol:
- A small percentage of elderly men will go in to acute retention of urine after routine inguinal hernia surgery.
- This may be caused by a combination of prostatic hypertrophy, combined with the anticholinergic effects of some anaesthetic reversal
agents. Pain also contributes.
- A brief (24 hours) of catherisation is usually all that is required.

Correct Answer. a

(181). Injection sclerotherapy can be successful in varicose veins less than :

a. 3 mm

b. 5 mm

c. 7 mm

d. 9 mm

Solution. (a) 3 mm Sol.


- Varicose veins may be managed by injection sclerotherapy or surgical excision or a combination of both techniques.
- Injection sclerotherapy can be successful in varicose veins less than 3 mm in diameter and in telangiectatic vessels.
- Sclerosing agents include hypertonic saline, sodium tetradecyl sulfate, and polidocanol. An elastic bandage is worn continuously for 3–5
days postsclerotherapy.
- Complications of sclerotherapy include allergic reaction, pigmentation, thrombophlebitis, DVT, and possible skin necrosis.

Correct Answer. a

Copyright © 2014 Delhi Academy of Medical Sciences, All Rights Reserved. 84/93
(182). The first diagnostic test in patients with suspected esophageal disease should be a:

a. Fluoroscopy

b. Endoscopy

c. X-ray soft tissue neck.

d. Barium swallow

Solution. (d) Barium swallow Sol.


- The first diagnostic test in patients with suspected esophageal disease should be a barium swallow including a full assessment of the
stomach and duodenum.
- Esophageal motility is optimally assessed by observing several individual swallows of barium traversing the entire length of the organ,
with the patient in the horizontal position.

Correct Answer. d

(183). Hiatal hernias are best demonstrated with the patient placed :

a. Supine

b. Prone

c. Left lateral

d. Right lateral

Solution. (b) Prone Sol. Hiatal hernias are best demonstrated with the patient prone because the increased intraabdominal pressure
produced in this position promotes displacement of the esophagogastric junction above the diaphragm.

Correct Answer. b

(184). Which of the given complication can occur following the use of a colonic segment for uninary diversion : -

a. Hyperchloremia,hypokalemia and metabolic acidosis

b. Hyperchloremia,hypokalemia and metabolic alkalosis

c. Hypochloremia,hypokalemia and metabolic alkalosis

d. Hypochloremia,hyperkalemia and metabolic alkalosis

Solution. (a) Hyperchloremia,hypokalemia and metabolic acidosis Sol: Metabolic abnormalities associated with urinary diversions.
1.Colonic Conduits → Hyperchloremic, Hypokalemic metabolic acidosis 2.Jejunal Conduits → Hypochloremic / hyponatremic metabolic
aciosis with hyper kalemia. 3.Gastric conduit→Hypochloremic hypokalemic metabolic alkalosis.

Correct Answer. a

Copyright © 2014 Delhi Academy of Medical Sciences, All Rights Reserved. 85/93
(185). 45 yrs old female tobacco chewer presents with 4.0 cms ulcerative growth on tongue;neither any lymph node is palpable nor any bony
involvement is there.Histopathology confirms the diagnosis of squamous cell carcinoma of tongue. Treatment of choice will be : -

a. Wide local excision and segmental mandibulectomy

b. Total glossectomy

c. Wide local excision

d. Hemiglossectomy

Solution. (d)Hemiglossectomy

Correct Answer. d

(186). TPN is indicated in all except:

a. Short bowel syndrome

b. Burns

c. Sepsis

d. Enterocutaneous fistula

Solution. (c) Sepsis Ref.: Sabiston - 168 Sol : TPN increases the risk of sepsis in two ways:
- Catheter related infection
- Increase translocation of GI bacteria

Correct Answer. c

Copyright © 2014 Delhi Academy of Medical Sciences, All Rights Reserved. 86/93
(187). A 35-year-old female developed feature of septicemia. Shock in form of hypotension and low urine output. She was being treated for colonic
necrosis. What will be the management?

a. IV fluid + dopamine

b. IV fluid only

c. Only dopamine

d. Antibiotic in high dose.

Solution. (a) IV fluid + dopamine Ref.: Harrison - 1605


Sol :

Correct Answer. a

(188). Greyish discoloration of flank seen in Acute Pancreatitis is referred to as?

a. Cullen’s Sign

b. Grey Turner Sign

c. Balance’s Sign

d. Alvaradao’s Sign

Solution. (b) Grey Turner Sign Ref.: Read the text below
Sol : - Grey Turner's sign refers to bruising of the flanks, the part of the body between the last rib and the top of the hip.
- The bruising appears as a blue discoloration,and is a sign of retroperitoneal hemorrhage, or bleeding behind the peritoneum, which is a
lining of the abdominal cavity.
- Grey Turner's sign takes 24–48 hours to develop, and can predict a severe attack of acute pancreatitis

Correct Answer. b

Copyright © 2014 Delhi Academy of Medical Sciences, All Rights Reserved. 87/93
(189). Most common site of esophageal perforation is?

a. Cervical region

b. Cardiac region

c. Mid esophagus

d. Lower esophagus

Solution. (a) Cervical region Ref.: Read the text below


Sol :
- Most esophageal perforations occur after endoscopic instrumentation for a diagnostic or therapeutic procedure, including dilation,
stent placement, and laser fulguration.
- Other iatrogenic causes that have been noted include difficult endotracheal intubation, blind insertion of a mini-tracheostomy, and
inadvertent injury during dissections in the neck, chest and abdomen.
- “Perforation of the oesophagus is usually iatrogenic (at therapeutic endoscopy) or due to ‘barotrauma’ (spontaneous performation) with
cervical region most commonly involved”

Correct Answer. a

(190). Most common cause of intestinal obstruction is?

a. Ileocaecal tuberculosis

b. Carcinoma colon

c. Intussusception

d. Adhesions

Solution. (d) Adhesions Ref.: Read the text below


Sol :
- With the routine elective repair of hernias, this cause has dropped to the third most common cause of small bowel obstruction in
industrialized countries.
- Adhesions secondary to previous surgery are by far the most common cause of small bowel obstruction today.
- Intra-abdominal adhesions related to prior abdominal surgery account for up to 75% of the cases of small bowel obstruction.

Correct Answer. d

(191). Antibiotic of choice for lymphedema is?

a. Penicillin

b. Amikacin

c. Metronidazole

d. Ceftazidime

Solution : --Not Available--

Correct Answer. a

Copyright © 2014 Delhi Academy of Medical Sciences, All Rights Reserved. 88/93
(192). The most preferred approach for pituitary surgery at the present time is

a. Transcranial

b. Transethmoidal

c. Transphenoidal

d. Transcallosal

Solution. (c) Transphenoidal Ref.: Read the text below Sol :


- The standard surgical approach for the majority of pituitary tumors is transphenoidal which is safer and better folerated than the
transcranial approach”
Most pituitary adenoma are approached through the nose via transphenoidal approach”.
- Transcranial approach is limited to the treatment of
- Suprasellar tumor that are tough and fibrous.
- Tumors that extend laterally in the middle fossa.
- Tumors that extend anteriorly or beneath the frontal lobes.

Correct Answer. c

(193). An old man who is edentubous developed squamous cell Ca in buccal mucosa that has infiltrated to the alveolus. Following is not
indicated in the treatment :

a. Radiotherapy

b. Segmental mandibulectomy

c. Marginal mandibulectomy involving removal of the outer table only

d. Marginal mandibulectomy involving removal of upper half of mandible

Solution. (c) Marginal mandibulectomy involving removal of the outer table only Ref.: Read the text below
Sol :
- In buccal Ca involving alveolus, marginal mandibulectomy with removal of inner table of tooth is indicated.
- Radiotherapy can be used as sole therapy in patient who are not suitable for surgery.
Mandibulectomy is of two types Marginal mandibulectomy :
- Partial excision of superior portion of the mandible in the vertical plane. The inner cortical surface and a portion of the underlying
medullary cavity is excised, the inferior half of the body remain intact thus preserving mandibular continuity.
- It is indicated when tumor lies with in 1 cm of mandible or when it touches the periosteum without evidence of direct bone invasion
Segmental mandibulectomy :
- Removal of entire segment of mandible, thus resulting in mandibular discontinuity and requires reconstructive procedures.

Correct Answer. c

(194). Stone formation is most common in due of:

a. Submandibular gland

b. Sublingual gland

c. Parotid gland

d. Lacrimal gland

Solution. (a) Submandibular gland Ref.: Read the text below


Sol :
- Stone formation is most common in duct of submandibular gland represent 80-90 percent of stone.
- Most of parotid duct stone are radiolucent while most off submandibular stones are radiopaque.
- Stones are formed by the deposition of calcium phosphate on the organic matrix matrix of mucin or cellular debris.
- Presenting feature is intermittent swelling of involved gland and pain due to obstruction to outflow of saliva.
- Stones in peripheral part of submandibular or parotid duct can be removed intraorally, while those at hilum require excision of gland.

Correct Answer. a

Copyright © 2014 Delhi Academy of Medical Sciences, All Rights Reserved. 89/93
(195). The most common site of the internal opening of a branchial fistula is at the –

a. Lateral nasopharyngeal wall

b. Fossa of Rosenmuller

c. Gingivo-labial sulcus

d. Tonsillar fossa

Solution. (d) Tonsillar fossa Ref.: Read the text below


Sol : Brachial fistula
- External opening : Situation in the lower third of the neck, near the anterior border of sternomastiod.
- Internal opening : Anterior aspect of posterior faucial pillar

Correct Answer. d

(196). Who got Nobel prize for his contribution in vascular Surgery?

a. Theodore Kocher

b. Alexis Carrel

c. Christian Bernard

d. Joseph Murray

Solution. 196:(b) Alexis Carrel


Ref: Read the text below

Correct Answer. b

Copyright © 2014 Delhi Academy of Medical Sciences, All Rights Reserved. 90/93
(197). Indication of Biopsy in renal mass is?

a. Metastasis and infective lesions

b. Oncocytoma and angiomylipoma

c. Adenoma and cystic adenoma

d. All cases of renal tumours confined within Gerotas fascia

Solution. (a) Metastasis and infective lesions


Ref: Read the text below

Correct Answer. a

(198). Staging used for RCC is?

a. Robsons

b. McFrlane

c. Scimada

d. Masaoka

Solution. (a) Robsons


Ref: Read the text below

Correct Answer. a

Copyright © 2014 Delhi Academy of Medical Sciences, All Rights Reserved. 91/93
(199). True about ileostomy output is

a. Sodium excretion is two to three times that of normal stool

b. Contents of ileostomy are alkaline

c. Usually it starts functioning in 24 hours

d. Uric Acid renal calculus formation is more common than cholelithiasis

Solution. (a) Sodium excretion is two to three times that of normal stool
Ref: Read the text below Sol:
- An ileostomy starts to function 48 to 72 hours after construction. A mature ileostomy produces between 400 and 700 mL of effluent per
day.
- This volume remains relatively constant for an individual. The contents are weakly acidic (pH 6.1 to 6.5).
- Sodium excretion is 60 to 120 mEq/day, which is two to three times higher than in normal feces.
- Cholelithiasis occurs in 30% and Renal stones in 10%

Correct Answer. a

(200). A 75 yrs old lady presented to ER with H/O chronic constipation, pain abdomen and obstipation for 1 day. Abdomen was grossly
distended and tympanetic on percussion. X ray abdomen showed “OMEGA sign”. Next step in management should be?

a. CECT abdomen.

b. Ba enema.

c. Flexible Sigmoidoscopy.

d. Exploratory laparotomy.

Solution. (c) Flexible Sigmoidoscopy


Ref: Read the text below
Sol:
- Here the diagnosis is sigmoid volvulus. Initial treatment for sigmoid volvulus is sigmoidoscopy to derotate the volvulus.
- Treatment of the sigmoid volvulus begins with appropriate resuscitation and in most cases involves nonoperative decompression.
- Patients with signs of colonic necrosis are not eligible for nonoperative decompression.
- Decompression can occur by placement of a rectal tube through a proctoscope or the use of a colonoscope.
- Often, a soft rectal tube can be inserted under direct vision through the twist of the volvulus while the patient is in the emergency room.
- If a rectal tube cannot be passed, detorsion of the volvulus with the colonoscope should be attempted.
- If detorsion of the volvulus cannot be accomplished with either a rectal tube or colonoscope, laparotomy with resection of the sigmoid
colon (Hartmann's operation) is required.

Correct Answer. c

Test Answer
1.(b) 2.(c) 3.(c) 4.(d) 5.(c) 6.(a) 7.(a) 8.(a) 9.(c) 10.(d)

11.(d) 12.(b) 13.(b) 14.(b) 15.(c) 16.(c) 17.(d) 18.(c) 19.(b) 20.(c)

21.(c) 22.(c) 23.(a) 24.(b) 25.(b) 26.(a) 27.(d) 28.(b) 29.(a) 30.(d)

31.(b) 32.(b) 33.(b) 34.(d) 35.(c) 36.(c) 37.(c) 38.(a) 39.(a) 40.(c)

41.(b) 42.(a) 43.(c) 44.(c) 45.(c) 46.(a) 47.(a) 48.(b) 49.(a) 50.(a)

51.(c) 52.(d) 53.(a) 54.(d) 55.(a) 56.(b) 57.(b) 58.(d) 59.(a) 60.(a)

61.(a) 62.(b) 63.(d) 64.(a) 65.(b) 66.(a) 67.(b) 68.(d) 69.(d) 70.(c)

71.(a) 72.(a) 73.(a) 74.(d) 75.(d) 76.(b) 77.(d) 78.(d) 79.(d) 80.(d)

81.(a) 82.(d) 83.(b) 84.(c) 85.(a) 86.(a) 87.(b) 88.(c) 89.(a) 90.(a)

91.(a) 92.(d) 93.(d) 94.(c) 95.(d) 96.(d) 97.(b) 98.(c) 99.(b) 100.(a)

101.(b) 102.(d) 103.(b) 104.(b) 105.(a) 106.(b) 107.(a) 108.(c) 109.(d) 110.(a)

111.(c) 112.(c) 113.(d) 114.(d) 115.(a) 116.(a) 117.(d) 118.(c) 119.(b) 120.(b)

121.(a) 122.(a) 123.(c) 124.(c) 125.(a) 126.(d) 127.(d) 128.(d) 129.(c) 130.(d)

131.(b) 132.(d) 133.(a) 134.(a) 135.(d) 136.(b) 137.(d) 138.(c) 139.(c) 140.(b)

141.(d) 142.(a) 143.(d) 144.(b) 145.(d) 146.(c) 147.(a) 148.(d) 149.(c) 150.(d)

Copyright © 2014 Delhi Academy of Medical Sciences, All Rights Reserved. 92/93
151.(a) 152.(a) 153.(b) 154.(a) 155.(b) 156.(a) 157.(b) 158.(c) 159.(a) 160.(c)

161.(d) 162.(d) 163.(c) 164.(b) 165.(b) 166.(d) 167.(c) 168.(d) 169.(d) 170.(a)

171.(b) 172.(a) 173.(c) 174.(d) 175.(a) 176.(a) 177.(b) 178.(b) 179.(b) 180.(a)

181.(a) 182.(d) 183.(b) 184.(a) 185.(d) 186.(c) 187.(a) 188.(b) 189.(a) 190.(d)

191.(a) 192.(c) 193.(c) 194.(a) 195.(d) 196.(b) 197.(a) 198.(a) 199.(a) 200.(c)

Copyright © 2014 Delhi Academy of Medical Sciences, All Rights Reserved. 93/93

You might also like